test 1

Pataasin ang iyong marka sa homework at exams ngayon gamit ang Quizwiz!

A home health nurse is working with a client who has chronic obstructive pulmonary disease. Which nursing diagnosis will take the highest priority for implementing client education? A) Impaired Gas Exchange B) Ineffective Breathing Pattern C) Anxiety D) Activity Intolerance

A) All of these nursing diagnoses are appropriate for the client who is experiencing chronic obstructive pulmonary disease, but the priority for the nurse to address is impaired gas exchange. If the client's oxygen level is too low, or the carbon dioxide level is too high, the client's life may be threatened. This client will also experience activity intolerance, anxiety, and, at times, ineffective breathing patterns, but the priority diagnosis is Impaired Gas Exchange.

Clinical manifestations of metabolic alkalosis are similar to signs of A) hypocalcemia. B) hypokalemia. C) hypercalcemia. D) hyperkalemia.

A) Manifestations of metabolic alkalosis result from decreased calcium ionization and are similar to those of hypocalcemia. They include numbness and tingling around the mouth, fingers, and toes; dizziness; Trousseau sign; and muscle spasm. Clinical manifestations of metabolic alkalosis are not similar to those of hypercalcemia or hyper- or hypokalemia.

The nurse is planning discharge teaching to a client with diabetes who has a large wound. Which is the priority action for the nurse prior to initiating teaching with this client? A) Asking the client to state what is known about the current dressing changes B) Teaching the client how to take blood sugars C) Assessing the client's ability to self-administer insulin D) Determining the client's reaction to having diabetes

A) Nurses need to provide client education that will ensure the client's safe transition from one level of care to another and make appropriate plans for follow-up education in the client's home. Discharge plans must include information about what the client has been taught before transfer or discharge and what remains for the client to learn to perform self-care in the home or other residence.

Traditional client records employ which of the following documentation systems? A) Source-oriented record B) Problem-oriented medical record C) PIE model D) Focus charting

A) The traditional client record is a source-oriented record, in which each individual or department makes notations in a separate section or sections of the client's chart. In many healthcare environments, this traditional system has been replaced by newer methods of documentation, such as the problem-oriented medical record (POMR), the PIE model, and focus charting.

The nurse is admitting a client suffering a panic attack to the behavioral health unit. Which clinical manifestations would indicate that the client's anxiety is at a panic level of severity? Select all that apply. A) Inability to focus B) Dilated pupils C) Feelings of doom D) Self-absorption E) Rapid speech

A, B, C) An inability to focus, dilated pupils, and a feeling of doom are clinical manifestations that a client could experience at the panic level of severity of anxiety. Self-absorption and rapid speech could indicate that a client is experiencing anxiety at a moderate level of severity.

The nurse is preparing to admit a client with acute pneumonia who is experiencing severe respiratory acidosis. Which treatments does the nurse anticipate as appropriate for this client? Select all that apply. A) Administer oxygen prn. B) Administer digoxin for heart failure. C) Encourage up to 3 L of fluids per day. D) Place in a prone position. E) Reposition frequently

A, C, E) The client with acute pneumonia and respiratory acidosis may require oxygen administration to improve gas exchange, increased fluid intake to thin secretions, and frequent repositioning to preventing the pooling of respiratory sections. There is not enough evidence to know whether the client is experiencing heart failure as a result of the acute pneumonia. The client should be placed in the Fowler or semi-Fowler rather than the prone position.

Which risk factors exhibited by the client presenting in the emergency department (ED) would place the client at risk for metabolic acidosis? Select all that apply. A) Abdominal fistulas B) Chronic obstructive pulmonary disease C) Pneumonia D) Acute renal failure E) Hypovolemic shock

A, D, E) Metabolic acidosis is rarely a primary disorder. It usually develops during the course of another disease; presence of abdominal fistulas, which can cause excess bicarbonate loss; acute renal failure; and hypovolemic shock. Chronic obstructive pulmonary disease and pneumonia place the client at risk for respiratory acidosis with the increased retention of carbon dioxide in the blood.

The nurse is caring for a client who has been admitted with persistent diarrhea lasting 3 days. Which are appropriate nursing diagnoses for this client during the acute phase of the illness? Select all that apply. A) Decreased Cardiac Output B) Ineffective Airway Clearance C) Overflow Urinary Incontinence D) Knowledge Deficit E) Risk for Injury

A, E) Metabolic acidosis decreases cardiac output by decreasing contractility, slowing the heart rate, and increasing the risk for dysrhythmias. The client with metabolic acidosis is also at risk for injury due to altered mental status. Appropriate nursing diagnoses during the acute phase of illness are Risk for Injury and Decreased Cardiac Output. The client may have a knowledge deficit, but this is not an appropriate nursing diagnosis during the acute phase of the illness. The client with metabolic acidosis is not at risk for Ineffective Airway Clearance or Overflow Urinary Incontinence.

A nurse recently attended a seminar that discussed the many threats to homeland security. As nurse manager of the emergency department, the nurse is responsible for planning for emergencies from bioterrorism. Which agents does the nurse include when planning for bioterrorism? Select all that apply. A) Anthrax B) Tuberculosis C) Cancer D) Flu E) Smallpox

A, E) Smallpox, anthrax, botulism, plague, viral hemorrhagic fevers, and tularemia are the agents that are of highest concern in regard to bioterrorism. Cancer, flu, and tuberculosis have not been developed into biological threats and would not kill the number of people that smallpox would.

The nurse is appointed to a clinical-administrative task force studying critical staffing issues and care delivery models for the hospital. Which evidence on the effects of different staffing choices and care delivery models should the task force consider? Select all that apply. A) Higher nurse-to-client ratios have been linked to a decrease in the amount of time clients are hospitalized. B) Shared governance is linked to a reduction in adverse outcomes. C) A higher proportion of registered nurses can reduce the risk of mortality in surgical clients. D) Research indicates that functional assignment of staff improves the likelihood of meeting clients' emotional needs. E) There is little or no research studying nursing ratios and client outcomes.

A. C) Higher nurse-to-client ratios have been associated with shorter lengths of stay and fewer complications. A recent study depicted a significant reduction in the risk of mortality in surgical clients when the proportion of registered nurses was higher. There is little research on care delivery models since it is more difficult to compare outcomes because each unit may employ slight variations of the model. Research continues to prove that higher nurse-to-client ratios result in fewer adverse outcomes for clients.

The nurse is caring for a client who has been admitted to the hospital for congestive heart failure. Which data collected during the nursing assessment indicate that the client is at risk for metabolic alkalosis? Select all that apply. A) The client takes furosemide (Lasix) daily. B) The client takes a baby aspirin once daily. C) The client takes metformin daily. D) The client frequently uses calcium carbonate (Tums) for acid indigestion. E) The client takes acetaminophen as needed for pain.

A. D) Excessive use of calcium carbonate and daily use of furosemide can cause metabolic alkalosis. Use of metformin is not associated with alkalosis. Overuse of aspirin can be associated with metabolic acidosis. Occasional use of acetaminophen is not associated with metabolic alkalosis.

The most common disorder that increases a client's risk for respiratory alkalosis is: A) a respiratory disorder. B) an anxiety disorder. C) a cardiovascular disorder. D) a congenital disorder.

B) Anxiety with hyperventilation is the most common cause of respiratory alkalosis. Therefore, anxiety disorders increase the risk for respiratory alkalosis. Respiratory disorders are more likely to lead to respiratory acidosis. Cardiovascular disorders and congenital disorders do not usually lead to respiratory alkalosis.

A young mother brings her children to the health department for routine immunizations. The mother is crying because she lost her job and fears that her asthmatic daughter may be denied appropriate healthcare because she is unable to pay for it. The nurse realizes that many clients are experiencing these fears. Which action by the nurse would have the greatest impact for all clients who experience this fear? A) Reminding the client that everything will be fine B) Becoming involved in the American Nurses Association (ANA) C) Moving to an area that needs healthcare workers D) Giving the client medications and supplies from the clinic at no charge

B) Because nurses are in a position to be closely associated with the needs of clients, nurses need to be involved in professional organizations that participate in educating legislators at the local and national levels. An example of such an organization is the American Nurses Association (ANA). For nurses to move to areas needing healthcare workers would not help with the rising cost of healthcare. Telling a worried parent that everything will be fine is giving the client potentially false hope. Giving clients supplies and medications from the clinic increases costs and will not help solve the overall problem with healthcare today.

The charge nurse assesses clients during a mass casualty incident (MCI) and transfers some to other units but discharges others to home. In planning for the admission of critically ill clients from the emergency department, to which nurses will the charge nurse assign the new clients when admitted to the unit? A) Nurses with risk-reduction knowledge B) Nurses with advanced assessment skills C) Nurses with impeccable ethics D) Nurses with exceptional self-care methods

B) During the admission of injured clients, the charge nurse would assign the new clients to those nurses with advanced assessment, technical, and communication skills. Advanced practice nurses who have received training in emergency and trauma care will have significantly greater responsibilities than nurses with less training. Nurses do need to have good ethics, sufficient risk-reduction knowledge, and appropriate self-care knowledge during emergency responses. However, during the admission of victims, the charge nurse would want the highly competent nurses caring for the new clients

The nurse is caring for a client who is reporting a pain level of 8 on a 0-to-10 numeric pain scale. The nurse administers the prescribed pain medication. When the nurse re-evaluates the client 1 hour later, the client is still reporting a pain level of 8. Which action by the nurse is appropriate at this time? A) Wait for the healthcare provider to make rounds to report the problem. B) Report to the healthcare provider by telephone. C) Increase the dosage of the medication. D) Include an entry in the nursing report indicating that the medication is ineffective

B) In this case, reporting to the healthcare provider by telephone is appropriate. The nurse would address the client's distress immediately and later include the event in the end-of-shift report to the oncoming nurse. The nurse cannot alter the dose of medication. Waiting for the provider to arrive could cause the client to experience a great deal of pain in the interim.

The nurse is caring for a 3-month-old infant who presented to the emergency department (ED) with fever, diarrhea, vomiting, and diaper rash over the past 48 hours. Which symptom puts this client most at risk for metabolic acidosis? A) Fever B) Diarrhea C) Vomiting D) Diaper rash

B) Infants are more susceptible to metabolic acidosis from diarrhea due to significant losses of bicarbonate in the feces. Vomiting is more likely to result in metabolic alkalosis from loss of stomach acids. Fever and diaper rash do not directly contribute to metabolic acidosis.

A compensatory mechanism that may indicate to a nurse that a client is experiencing metabolic acidosis includes: A) headache. B) Kussmaul respirations. C) vomiting. D) decreased level of consciousness.

B) Kussmaul respirations are deep and rapid respirations that are a compensatory mechanism during metabolic acidosis. Headache, vomiting, and decreased level of consciousness are all clinical manifestations of metabolic acidosis, but they are not compensatory mechanisms that the body uses to maintain acid-base balance during metabolic acidosis.

A nurse is preparing to discharge a client who experienced a myocardial infarction. The client will have to make many lifestyle changes, and the nurse is providing instruction on how to implement a heart-healthy lifestyle. Which is the best description of the client education the nurse is presenting to this client? A) Dependent function of nursing that needs a healthcare provider's order to implement B) Important independent nursing function C) Activity nurses begin to learn after training on the job D) Way to establish the client's dependence on the nurse

B) State nurse practice acts include client teaching as a function of nursing, thereby making teaching a legal and professional responsibility. Nurses seek to help clients manage their health independently. Nurses begin to learn about teaching during their training. Nurses are not dependent on healthcare providers when determining the learning needs of the client.

What are the four steps of the SBAR communication technique? A) Scenario, Basics, Analysis, and Reaction B) Situation, Background, Assessment, and Recommendation C) Scenario, Background, Analysis, and Recommendation D) Situation, Basics, Assessment, and Reaction

B) The SBAR technique provides a framework for safe, efficient communication between members of the healthcare team. In the first or "Situation" step, the nurse provides a concise statement of the problem. In the second or "Background" step, the nurse relates information relevant to the situation. In the third or "Assessment" step, the nurse provides an analysis and consideration of options. Finally, in the fourth or "Recommendation" step, the nurse provides a recommendation based on the relevant evidence.

The nurse is planning care for the client with Cushing syndrome who has been admitted for complications related to the disease process. Which intervention should the nurse plan for this client to improve the impaired gas exchange? A) Monitor serum electrolytes. B) Schedule nursing activities to allow for periods of rest. C) Assess input and output accurately. D) Administer IV fluids per practitioner order.

B) The client with Cushing syndrome is at risk for developing severe metabolic alkalosis that causes hypoxemia and limits energy reserves. Spacing nursing activities throughout the day allows the client ample rest time. The other interventions are aimed at the deficient fluid volume that may occur with metabolic alkalosis.

A nurse is caring for a client who had a total hip replacement 14 days ago. The client is preparing for discharge in a few days. The nurse facilitates a care conference with the primary healthcare provider, occupational therapist, physical therapist, and the client and family to develop a plan of care for the client prior to discharge. Which roles are being demonstrated by this nurse? Select all that apply. A) Coordinator B) Collaborator C) Differentiated practitioner D) Team leader E) Expert

B) The nurse is solving problems and planning care for the client by coordinating and collaborating with the other health team members who will participate in client care. All members of the team may be experts, but the level of expertise is not the role the nurse is exhibiting. The nurse is not responsible for how the members of the health team perform their jobs as a team leader would be. Differentiated practice involves nursing care from all levels of education and is not illustrated in this scenario.

A client begins to hyperventilate after learning that a breast biopsy was positive for cancer. After a few minutes, the client loses consciousness. Which action by the nurse is the priority? A) Begin cardiopulmonary resuscitation. B) Raise the side rails on the bed. C) Notify the physician. D) Insert an intravenous access device.

B) The nurse should protect the client from injury. If hyperventilation continues to the point where the client loses consciousness, respirations will return to normal, as will acid-base balance. The nurse should ensure the client's safety and raise the side rails on the bed. The client does not need cardiopulmonary resuscitation. The physician may need to be notified; however, the client's safety is a priority. The client is not critically ill, and an intravenous access device is not indicated at this time.

Acid-base balance is critical to homeostasis and cellular function. One measurement of acid-base balance is the pH of the blood. Which ion is measured to determine the pH? A) Cl- B) H+ C) Na+ D) HCO3

B) The pH measures the concentration of hydrogen ions (H+) in the body. Sodium (Na+) and chloride (Cl-) concentrations are not related to pH. Bicarbonate (HCO3) is a weak base that is used as a buffer to help maintain the proper pH, but it is not used to measure pH.

A client with pneumonia develops respiratory acidosis. Based on provider's orders, which medications should the nurse prepare to administer to this client? Select all that apply. A) The loop diuretic furosemide (Lasix), 20 mg by mouth twice a day B) The antibiotic amoxicillin, 1 gram intravenous every 6 hours C) The bronchodilator albuterol, inhaler 2 puffs every 4 hours D) The anxiolytic diazepam (Valium), 2 mg by mouth at bedtime for sleep E) Potassium chloride 20 mEq in 100 mL 0.9% normal saline intravenous every day

B, C) Bronchodilator drugs such as an albuterol inhaler may be administered to open the airways, and antibiotics such as amoxicillin may be prescribed to treat respiratory infections. Benzodiazepines such as diazepam are central nervous system depressants and would adversely affect this client's respiratory rate, adversely affecting respiratory acidosis. Potassium chloride is indicated in the treatment of metabolic alkalosis.

A nurse manager is educating staff nurses about the types and frequency of documentation required for clients being cared for in long-term care facilities. These requirements originate from which of the following laws and regulatory bodies? Select all that apply. A) Problem-Oriented Medical Record (POMR) Act B) Omnibus Budget Reconciliation Act (OBRA) C) Health Care Financing Administration (HCFA) D) Minimum Data Set (MDS) Act E) American Recovery and Reinvestment Act (ARRA)

B, C, E) Long-term care facilities must comply with documentation requirements set forth in the Omnibus Budget Reconciliation Act (OBRA) of 1987 and the American Recovery and Reinvestment Act (ARRA) of 2009, as well as with regulations established by the Health Care Financing Administration (HCFA). The POMR is a system of medical recordkeeping, not a law or regulatory body. The Minimum Data Set (MDS) is an effort to establish standards for collecting standardized, essential nursing data for inclusion in computer databases. Although established under OBRA, the MDS is not a specific law or regulatory body.

The nurse is helping with discharge planning for a client who needs extensive rehabilitation and is on a complicated medication schedule. The client and spouse currently live with their eldest son, and the family is very involved in the client's care. The nurse would want to coordinate the client's daily care by including which priority individuals? Select all that apply. A) Pharmacist B) Client's children C) Primary healthcare provider D) Social worker E) Client's spouse

B, E) Effective discharge planning necessitates health team conferences and family conferences and gives the client, family, and healthcare professionals the opportunity to plan care and set goals. Involving the client's spouse and children would be important in this situation because of the complexity of the client's situation. The physician, pharmacist, and social worker are important for the coordination of care in general, but they would not be involved in daily care in most circumstances.

The client is admitted to the emergency department (ED) with symptoms of a panic attack, including hyperventilation. Based on this data, the nurse plans care for which health problem? A) Hypoventilation B) Vomiting C) Respiratory alkalosis D) Memory loss

C) Anxiety disorders increase the risk for the acid-base imbalance respiratory alkalosis, due to hyperventilation that accompanies anxiety and panic attacks. The client with anxiety does not necessarily have vomiting or memory loss as risk factors. Anxiety and panic attacks will lead to hyperventilation, not hypoventilation.

The nurse is caring for a comatose client with metabolic acidosis. For which intervention will the nurse need to collaborate when caring for this client? A) Measuring vital signs B) Measuring intake and output C) The client's recent eating behaviors D) Identifying current oxygen saturation level

C) For clients in severe distress, family members may need to be consulted for critical information such as recent eating habits and history of vomiting. Measuring vital signs, measuring intake and output, and identifying current oxygen saturation level are independent nursing actions.

Free-floating anxiety is often connected to what stimulus? A) Elevators B) Airplanes C) No specific stimulus D) Water

C) Free-floating anxiety is characterized by excessive worry that is hard to control and whose focus may shift from moment to moment. Free-floating anxiety is anxiety that is not connected to a specific stimulus. Anxiety caused by elevators, airplanes, and water is linked to a specific stimulus.

The manager of a small clinic has cross-trained the nurses to perform electrocardiogram (ECG) testing, phlebotomy, and some respiratory therapy interventions. This clinic is providing client-focused care. Which of the following actions shows this delivery model in action? A) Many disciplines collaborate to provide client care. B) Client care is carefully managed to control costs. C) If a client complains of breathing difficulty, nurses concentrate on respiratory therapy for that client. D) Client progress is efficiently tracked.

C) In client-focused care, the needs of the client, physical or emotional, as expressed by the client drive care decisions. Interdisciplinary collaboration, careful cost management, or efficient tracking of client progress are not distinguishing features of this model.

The nurse is planning care for an older client with respiratory acidosis. Which intervention should the nurse include in this client's plan of care? A) Administer prescribed intravenous fluids carefully. B) Administer intravenous sodium bicarbonate. C) Maintain adequate hydration. D) Reduce environmental stimuli.

C) In respiratory acidosis, there are a drop in the blood pH, a reduced level of oxygen, and retention of carbon dioxide. The body needs to be well-hydrated so that pulmonary secretions can be removed to improve oxygenation. Careful administration of intravenous fluids is important in the older client with metabolic alkalosis because older clients are at risk because of their fragile fluid and electrolyte status. Sodium bicarbonate is indicated in the treatment of metabolic acidosis. Reducing environmental stimuli would be appropriate for the client with respiratory alkalosis.

A nurse is applying for a job as a case manager for a managed care insurance organization. Which responsibility is associated with this role? A) Providing home visits to clients B) Independent treatment planning C) Coordinating client care over time D) Approving treatment decisions

C) Insurance-based case management is a labor-intensive activity that is provided typically by telephone, not home visits. Case management involves interdisciplinary teams that assume collaborative responsibility for planning and assessing needs, not independent planning. Case management involves coordinating, implementing, and evaluating care for groups of clients from preadmission through discharge or transfer and recuperation. Approving treatment decisions is not a case management role

A client has been vomiting for several days. The nurse knows that the client is at risk for metabolic alkalosis because gastric secretions have which characteristic? A) Gastric secretions are green in color. B) Gastric secretions are alkaline. C) Gastric secretions are acidic. D) Gastric secretions have a foul smell.

C) Metabolic alkalosis due to loss of hydrogen ions usually occurs because of vomiting or gastric suction. Gastric secretions are highly acidic (pH 1-3). When these are lost through vomiting or gastric suction, the alkalinity of body fluids increases. This increased alkalinity results from the loss of acid and from selective retention of bicarbonate by the kidneys as chloride is depleted. Gastric secretions are not alkaline. The color and odor of gastric secretions have no influence on the development of metabolic acidosis.

A nurse is working in a neonatal intensive care unit (NICU). The nurse wants to teach a mother of a premature baby how to give her baby a bath. Which statement by the mother reflects a readiness to learn? A) "You'll give us written instructions before we go home, correct?" B) "When my baby is just a little bigger, I'll feel more comfortable giving him a bath." C) "I want to make sure my husband is here, in case I don't hear everything that's said." D) "I'm so afraid I'll hurt my baby with all these tubes and wires."

C) Readiness to learn is the demonstration of behaviors or cues that reflect a learner's motivation, desire, and ability to learn at a specific time. The client who wants her husband involved is demonstrating motivation and willingness to learn. Statements about fear of the situation need to be addressed so that the fear will not inhibit the learning process. Wanting to wait until discharge or at least until the baby is older reflects uncertainty and possibly fear and should be addressed before learning can occur.

The nurse is discharging a client diagnosed with general anxiety disorder (GAD). The client is prescribed a selective serotonin reuptake inhibitor (SSRI). Which statement made by the client would indicate to the nurse a need for further education? A) "This medicine could make me feel like I have the jitters." B) "I may experience some nausea while on this medication." C) "My doctor will start me off on a high dose and then decrease the dose." D) "This medicine alters the levels of the neurotransmitter serotonin in the brain."

C) SSRIs are generally started at low doses and then increased as their effectiveness becomes apparent; therefore, this statement made by the client is inaccurate and indicates a lack of understanding and the need for further teaching. The other statements are accurate so do not require further teaching.

A nurse is completing discharge teaching for a client who is hospitalized for total hip replacement. The client asks the nurse why there is a case manager involved and expresses confusion about who is in charge. The client states, "I thought my doctor manages my care." Which is the best response by the nurse? A) "No, I manage your care." B) "You are correct; the doctor is responsible for managing your care." C) "A case manager coordinates everyone involved in your care to ensure your needs are met." D) "The case manager delegates your care to the nurse."

C) The case manager is responsible for ensuring that all the client's healthcare needs are met in a cost-effective manner. The nurse may be a case manager; however, a staff nurse is not the most likely individual in the hospital setting to be the case manager. An agency usually has several case managers who collaborate with nursing, the physician, and any other departments involved in the care of the client. A physician does not participate in care by being a case manager. Case managers coordinate disciplines of care for the client and do not delegate any care to other professionals.

The nurse is preparing to discharge a client with congestive heart failure on furosemide (Lasix). The nurse determines that teaching has been effective if the client makes which statement? A) "I will use only sodium bicarbonate as my antacid." B) "I will restrict my intake of fluids." C) "I will use potassium supplements while I am taking Lasix." D) "I will take antacids only for my gastric discomforts."

C) The client on furosemide (Lasix) may lose excess potassium, disposing the client toward metabolic alkalosis. The client is taught to refrain from the use of sodium antacids when prone to metabolic alkalosis. The client should consult with the primary care provider for gastric distress rather than self-medicate. The client who is prone to metabolic alkalosis is likely to have fluid deficits and would not be instructed to restrict fluids.

The nurse is instructing a client with a history of acidosis on the use of sodium bicarbonate. Which client statement indicates that additional teaching is needed? A) "I should contact the doctor if I have any gastric discomfort with chest pain." B) "I need to purchase antacids without salt." C) "I should use the antacid for at least 2 months." D) "I should call the doctor if I get short of breath or start to sweat with this medication."

C) The client should be instructed to immediately contact the primary healthcare provider if gastric discomfort occurs with chest pain or if dyspnea or diaphoresis occurs. The client should be instructed to use non-sodium antacids to prevent the absorption of excess sodium or bicarbonate into systemic circulation and to not use any bicarbonate antacid for longer than 2 weeks.

The nurse is caring for the client with a history of anxiety who is experiencing chest pain, palpitations, and dyspnea. Which intervention would be a priority for this client? A) Providing educational material for the client's medical diagnosis B) Ordering a regular diet for the client C) Reassuring the client that symptoms will resolve D) Asking Respiratory Therapy to set up a mechanical ventilator

C) The client will require reassurance from the nurse that the symptoms being experienced are not those of a heart attack and that the symptoms will resolve when the breathing pattern returns to normal. Ordering the diet and instructing the respiratory therapist are done by the healthcare provider. Providing teaching for the client becomes a priority when the client is recovering from the illness.

While reviewing laboratory results, the nurse notes that a client's potassium level is 2.8 mEq/L and chloride level is 100 mEq/L. Based on this data, which intervention does the nurse plan for this client? A) Preparing to administer 0.9% sodium chloride infusion B) Measuring for nasogastric tube insertion C) Discussing potassium chloride replacement therapy with the healthcare provider D) Reviewing implications of transfusing with ammonium chloride

C) Treatment of metabolic alkalosis includes restoring normal fluid volume and administering potassium chloride. The potassium restores serum and intracellular potassium levels, allowing the kidneys to conserve hydrogen ions more effectively. Because the chloride level is within normal limits, an infusion of 0.9% sodium chloride is not indicated. A nasogastric tube is not indicated for this client. There is not enough information to support the use of ammonium chloride for this client, as it is indicated to treat severe metabolic alkalosis.

Upon entering a room, the nurse quickly scans the environment and then immediately assesses the client for manifestations of metabolic acidosis. Which did the nurse observe to precipitate this client assessment? A) Client sleeping with the head of the bed flat B) Half of the client's lunch tray uneaten C) One formed stool in the bedside commode D) 2000 mL of intravenous 0.9% normal saline infused in 2 hours

D) Excessive infusions of chloride-containing intravenous fluids can precipitate metabolic acidosis. The head of the bed's being flat might influence a client's oxygenation status; however, the client was not demonstrating a change in respiratory depth or rate. A reduction in oral intake does not cause metabolic acidosis. Eating half of a meal tray is not the same as starvation. Diarrhea can lead to the development of metabolic acidosis. One formed stool would not cause the nurse alarm.

A nurse is caring for a child who is hospitalized for an exacerbation of asthma. The nurse is preparing discharge teaching, as the client will be going home on nebulizer treatments and an inhaler. The client and her family members, who are recent immigrants to the United States, speak little English. In addition to enlisting an interpreter to help with the language barrier, what should be a priority for the nurse in developing a teaching plan? A) Provide written instructions before discharge. B) Make sure the parents can set up the treatments for their child. C) Make sure the child comes back for the follow-up appointment. D) Address any healing beliefs the family has.

D) Providing an interpreter to assist with communication is extremely important in this situation. However, if the prescribed treatment conflicts with the client/family's cultural healing beliefs, the client may not be compliant with the recommended treatments. To be effective, nurses must deal directly with any conflicts and differing values held by the client's parents. It is also important to provide written material and assess the psychomotor skills of the child, but the first priority is ascertaining any belief conflicts that may interfere with the treatment and cause the parents to resist the prescribed treatment or bringing the child back for a follow-up appointment.

The nursing student is planning an educational program for a school project. The program is focusing on cancer detection education for a community group. What should the nursing student plan to include in order to address the various learning styles of the target group? A) A lecture using many examples for each learning need B) Multicolored brochures with bright colors C) A game board with client matching terms D) Audiovisuals, examples, group discussions, and activities

D) When teaching a group, use strategies to address visual, auditory, manipulative, group, and problem-solving types of learners. Using different techniques and a variety of activities is a good way to match the various learning styles of group participants. Multicolored brochures would only address those learners who learn in the visual mode. Lecture may not meet the needs of the entire group. Games are a useful teaching tool but necessarily not useful when addressing a large group of individuals with varying learning styles.

A client is brought to the emergency department (ED) after passing out in a local department store. The client has been fasting and has ketones in the urine. Which acid-base imbalance would the nurse expect to assess in this client? A) Metabolic acidosis B) Respiratory alkalosis C) Metabolic alkalosis D) Respiratory acidosis

A) A client who is fasting is at risk for development of metabolic acidosis. The body recognizes fasting as starvation and begins to metabolize its own fatty acids into ketones, which are metabolic acids. Starvation would not result in respiratory acidosis or alkalosis or in metabolic alkalosis.

The nurse is preparing to analyze a client's arterial blood gas results. List the steps in the order that the nurse should follow when analyzing this laboratory test. 1. Look at the PaCO2. 2. Look at the pH. 3. Evaluate the relationship between pH and PaCO2. 4. Look for compensation. 5. Evaluate the pH, HCO3, and base excess for a possible metabolic problem. 6. Look at the bicarbonate. 7. Evaluate oxygenation

2, 1, 3, 6, 5, 4, 7 Explanation: 1. The second step is to look at the PaCO2. If the PaCO2 is <35, then more carbon dioxide is being exhaled. If the PaCO2 is >45, then more carbon dioxide is being retained. 2. The pH is the first step and is analyzed to determine if acidosis or alkalosis is present. A pH of <7.35 is acidosis. A pH of >7.45 is alkalosis. 3. The third step is to evaluate the relationship between the pH and the PaCO2. This relationship could indicate a respiratory problem. If the pH is acidotic and the carbon dioxide level is greater than 45, then the client could be experiencing respiratory acidosis. If the pH is alkalotic and the carbon dioxide level is below 35, then the client could be experiencing respiratory alkalosis. 4. The sixth step is to look for compensation. Two things can occur in renal compensation. In respiratory acidosis, the kidneys retain HCO3 to buffer the excess acid, so the HCO3 is >26 mEq/L. In respiratory alkalosis, the kidneys excrete HCO3 to minimize the alkalosis, so the HCO3 is <22 mEq/L. Two things can also occur in respiratory compensation. In metabolic acidosis, the rate and depth of respirations increase, increasing carbon dioxide elimination, so the PaCO2 is <35 mmHg. In metabolic alkalosis, respirations slow and carbon dioxide is retained, so the PaCO2 is >45 mmHg. 5. The fifth step is to evaluate the pH, HCO3, and base excess for a possible metabolic problem. If the pH is <7.35, the HCO3 is <22 mEq/L, and the BE is less than −3 mEq/L, then low bicarbonate levels and high H+ concentrations are causing metabolic acidosis. If the pH is >7.45, the HCO3 is >26 mEq/L, and the BE is greater than +3 mEq/L, then high bicarbonate levels are causing metabolic alkalosis. 6. The fourth step is to look at the bicarbonate level. If the bicarbonate level is <22, then the levels are lower than normal. If the bicarbonate level is > 26, then the bicarbonate levels are higher than normal. 7. The final step is to evaluate oxygenation. If the PaO2 is <80 mmHg, then the client is experiencing hypoxemia and possible hypoventilation. If the PaO2 is >100 mmHg, then the client is hyperventilating.

The nurse is preparing to teach a client with type 1 diabetes mellitus on the mechanism behind the development of ketoacidosis. List the order in which the nurse should provide this information. 1. Production of lactate and hydrogen ions 2. Tissue hypoxemia 3. Breakdown of fatty tissue 4. Reduction in intracellular glucose 5. Fatty acids converted to ketones

2, 1, 4, 3, 5 Explanation: Lactic acidosis develops due to tissue hypoxia and a shift to anaerobic metabolism by the cells. Lactate and hydrogen ions are produced, forming lactic acid. Starvation or lack of insulin leads to intracellular starvation of glucose. The lack of glucose or insulin to move glucose into the cells, causing the body to break down fatty tissue to meet metabolic needs. When fatty acids are broken down, these acids are converted to ketones, leading to the development of ketoacidosis.

The nurse taking care of a client in the clinic notes that the client comes in twice per week, but never seems to have anything majorly wrong. Which is a benefit of providing education about self-care to this client? A) The client's perception of need will change, promoting appropriate and timely healthcare. B) The client will continue to come to the clinic seeking help until someone helps the client. C) The client will seek affordable insurance by which to gain appropriate healthcare. D) The client will learn to seek healthcare services from legitimate sources.

A) A client's perceived need for healthcare services can be a barrier to access. It may cause clients to seek care that is unnecessary or care that is necessary but provided at an inappropriate and often more expensive place of service than needed. The nurse can change clients' perception of need (thereby promoting appropriate and timely healthcare) by managing client care, teaching adults about self-care and the care of their children, and teaching clients when and how to access appropriate care. An educated client will not continue to come to the clinic when there is no need. The issue here is not the need for affordable insurance or the inability to recognize legitimate sources of care

The nurse is participating on a local council as an advisor regarding community needs during an emergency. Which recommendation regarding community needs during an emergency that the nurse might include when advising the council would be involved in the emergency response phase? A) A coordinated emergency preparedness plan B) Assembling disaster kits C) Programs to restore the community D) The identification of potential hazards to the community

A) A coordinated response to emergencies occurs during the emergency response phase. Although a comprehensive disaster plan is developed in the preparedness phase, it is enacted in the emergency response phase. During the mitigation phase of the emergency response, the community identifies the potential hazards and takes measures to prevent or minimize the emergency. Assembling disaster kits is part of the preparedness phase. Programs to restore the community are part of the recovery phase.

The nurse on third shift is handing off clients to the nurse on first shift. Which of the following statements is most important for the third shift nurse to report during this handoff? A) "The client in room 312 is complaining about a headache unrelieved by pain medication. I am awaiting a call from the physician for orders." B) "The client in room 313 ate a full meal several hours ago and is currently sleeping peacefully." C) "The client in room 315 received an enema at 2100." D) "The client in room 311 was transferred from room 212."

A) A good change-of-shift report is concise and does not elaborate on background data or routine care, such as enema administration, meal consumption, or past client transfers. Rather, an effective change-of-shift report highlights significant changes in or concerns with a client's condition (e.g., persistent unrelieved headache), as well as any pending orders or requests for orders related to these changes or concerns.

A client has been admitted with chronic obstructive pulmonary disease. Diagnostic tests have been ordered. Which of the tests will provide the most accurate indicator of the client's acid-base balance? A) Arterial blood gases (ABGs) B) Pulse oximetry C) Sputum studies D) Bronchoscopy

A) ABGs are done to assess alterations in acid-base balance caused by respiratory disorders, metabolic disorders, or both. A bronchoscopy provides visualization of internal respiratory structures. Sputum studies can provide specific information about bacterial organisms. Pulse oximetry is a noninvasive test that evaluates the oxygen saturation level of blood.

The nurse has a 7-year-old client recovering from partial-thickness burns to the arms and hands. This client has shown sensitivity to loud noises and bright lights, and at times if she is overstimulated she won't speak to or look at anyone but her parents until she calms down. The nurse considers the best teaching environment for this client to be the A) client's room. B) pediatric ward waiting area. C) hospital cafeteria. D) pediatric ward play area.

A) Be sure all teaching interventions are implemented in a safe environment using a calm approach, and take care to address any concerns or fears of the child or parent/caregiver. In this client's case, the waiting or play areas for the pediatric ward are likely to be busy places and brightly lit. The cafeteria is also likely to be too loud and bright. The client's room, where the nurse can control to a greater degree the amount of light and noise, is best for teaching this client.

The nurse is providing care to a client diagnosed with end-stage renal disease. When organizing a care plan conference for this client, whom should the nurse invite to participate? A) The client's family members B) A psychiatrist C) An oncologist D) The hospital CEO

A) Care plan conferences allow for collaborative reporting among the healthcare professionals who provide care to a client. They are most often used for clients who have complex care needs. During the conference, the client's healthcare providers discuss possible solutions to client problems. The choice of healthcare professionals who are invited to attend the conference is based on the needs of the client; given this client's diagnosis, it is unlikely that an oncologist or psychiatrist would be part of the healthcare team. However, family members are an important part of the care plan conference, especially for clients who are unable to advocate for themselves.

A school-age client is admitted to the hospital with respiratory acidosis. Which chronic lung illness in the client's health history does the nurse suspect is causing the current diagnosis? A) Cystic fibrosis B) Aspiration C) Hyperthyroidism D) Pneumonia

A) Chronic lung disease such as asthma and cystic fibrosis put the child at risk for respiratory acidosis. Pneumonia and aspiration are both acute lung conditions. Hyperthyroidism is a disorder that results in metabolic acidosis.

A client with what level of anxiety would be most receptive to learning tools that would help the client recognize triggers? A) Mild B) Moderate C) Severe D) Panic

A) Clients with mild anxiety would be able to learn about how to recognize triggers. Clients with higher levels of anxiety may be unable to learn new tools and would first need other interventions to help reduce their level of anxiety before they could learn how to recognize triggers for anxiety.

The nurse knows that communication among healthcare team members is essential during mass casualty events (MCEs). Which is essential when communicating under these circumstances? A) Providing concise, accurate, and timely information B) Preparing for ethical challenges C) Documenting to prevent legal issues D) Coordinating care between management and clinicians

A) Communication among the various emergency team members must be concise, accurate, and timely during an MCE. Nurses must use their knowledge to foster better communication. Nurses face ethical and legal issues associated with the provision of care in MCEs, and they are also challenged as they decide what care to provide to patients; however, these issues do not directly affect communication even though they are affected by communication.

Which theory of learning holds that knowledge acquisition is the ongoing assimilation and accommodation of new experiences and interpretations? A) Constructivist B) Behaviorist C) Social learning D) Cognitive

A) Constructivist theory holds that knowledge acquisition is the ongoing assimilation and accommodation of new experiences and interpretations. In behaviorist theory, learning is thought to occur when an individual's response to a stimulus is either positively or negatively reinforced. In social learning theory, learning primarily results from instruction and observation. In cognitive learning theory, learning involves the processes of acquiring, processing, and using new information.

A nurse working in an outpatient OB-GYN clinic is asked to assume the case management for the clinic's adolescent clients. Which skills will the nurse need to perform this work? A) Critical thinking B) Physical assessment C) Relaxation training D) Accounting

A) Critical thinking skills are crucial to the development of a well-coordinated care plan and its execution. Relaxation training skills and accounting skills are not a part of care coordination. Although the nurse may need to assess the plan of care and make adjustments as needed, the nurse in the role of case manager may not be required to perform physical assessments on the client.

A community health nurse runs a clinic that provides health screening mainly to Mexican American and Filipino American clients. The nurse wants to have a class on smoking cessation for interested adults of this group. What action should the nurse take to adjust to their time orientation? A) Organize the instructions around short-term objectives. B) Mail letters ahead of time to make sure clients are informed about the upcoming class. C) Make posters and place them in areas of the community frequented by these groups. D) Make sure that the classes are held at specific times.

A) Cultures with a predominant orientation to the present include the Mexican American, Navajo Native American, Appalachian, Eskimo, and Filipino American cultures. Preventing future problems may be less significant for these clients than for others, so teaching prevention may be more difficult. In such instances, the nurse can emphasize preventing short-term problems rather than long-term problems. Schedules have to be very flexible in present-oriented societies. Time constraints are not significant for cultures that are oriented to the present, so advertising about specific classes may not be effective. The nurse must be quite flexible, treat the culture's beliefs with respect, and not expect that cultural practices will change to reflect the nurse's needs.

During the physical examination of a client who took a fall that fractured his hip, the nurse notices an impairment of the client's hearing, but that the client's visual acuity and motor function do not seem to be impaired. The client answers questions very precisely and readily grasps the meaning of everything the nurse says when the client can face the nurse. When teaching this client, the nurse should make it a priority to A) make verbal instructions face to face with the client. B) provide only written instructions. C) use only visual media. D) use only physical demonstrations with written instructions.

A) During the physical examination, the nurse may use findings to evaluate learning needs. In this client's case, verbal instructions are fine to use if the client is in a position to see the nurse's lips move. Written instructions, visual media, and physical demonstrations also might be useful for this client when indicated, but not exclusively.

The nurse manager is discussing the preparedness phase of a revised emergency management plan for the emergency department. The American Nurses Association (ANA) is a resource the nurse manager can use to help nurses understand which of the following during an emergency response? A) The ethics of emergency response B) The procedure for working in a hot zone during a hazardous material incident C) The role of the incident commander in a disaster response D) The best means of communicating with agencies such as the Centers for Disease Control and Prevention (CDC) during a crisis

A) During the preparedness phase, individual nurses must gain an understanding of their expected roles in an emergency and prepare for them. Because nurses will be required to allocate scarce resources and supplies and make unbelievably difficult client care decisions, they must understand the ethics associated with such choices. The ANA is a good source of information to guide nurses' understanding of their roles and possible consequences. Nurses must be aware of their employer's response plans and have a sense of how their state and local community will operate during an emergency. The ANA is not a primary source of information for working in hot zones, the role of the incident commander, or the best means of communicating with agencies such as the CDC.

A client with injuries from a motor vehicle crash is intubated for respiratory support. The nurse notes that the client is fighting the ventilator and attempting to pull out the endotracheal tube. What should the nurse do to reduce this client's risk of developing respiratory alkalosis? A) Administer a sedative as prescribed. B) Apply wrist restraints. C) Teach the client to take slow, deep breaths. D) Discuss removing the endotracheal tube with the healthcare provider.

A) For a client being mechanically ventilated, the only way to reduce rapid respirations might be to provide a sedative. Applying wrist restraints to a client who is demonstrating anxiety with an endotracheal tube might increase the client's anxiety. The client is being mechanically ventilated, which means there is a problem with maintaining the airway. The client will not be able to take slow, deep breaths at this time. The reason for the endotracheal tube is to maintain the client's airway after chest trauma. Removing the tube could lead to a collapse of the airway and a life-threatening situation.

If a change in acid-base balance is due to hypoventilation or hyperventilation, the nurse will need to primarily focus on which concept related to acid-base balance? A) Oxygenation B) Perfusion C) Cognition D) Stress and coping

A) Hypoventilation and hyperventilation are related to oxygenation. Respiratory rate helps regulate carbon dioxide pressures, which can contribute to acidosis or alkalosis. The nurse can help reverse respiratory acidosis or alkalosis by helping the client control their respiratory rate to restore normal oxygenation. Perfusion, cognition, and stress and coping do not directly relate to hypo- or hyperventilation.

A client is admitted to the unit with chronic obstructive pulmonary disease. Blood gas analysis indicates respiratory acidosis. Based on this data, the nurse plans care based on which priority diagnosis? A) Impaired Gas Exchange B) Ineffective Airway Clearance C) Impaired Mobility D) Anxiety

A) Impaired Gas Exchange is the priority nursing diagnosis for the client with respiratory acidosis. Interventions are aimed at restoring effective alveolar ventilation and gas exchange. Anxiety and Ineffective Airway Clearance are both appropriate nursing diagnoses but not priority for the client with respiratory acidosis. There is no evidence to support the nursing diagnosis Impaired Mobility for this client

The nurse knows that Congress enacted the Emergency Medical Treatment and Labor Act (EMTALA) of 1986 to prevent which action by emergency services? A) Refusing to treat uninsured clients B) Servicing suburban clients only C) Stopping the poor from using emergency services as primary care D) Providing free examinations to the poor

A) In 1986, Congress enacted the Emergency Medical Treatment and Labor Act (EMTALA) to ensure public access to emergency services regardless of ability to pay. Prior to the enactment of this law, providers of emergency services often refused to treat clients who were uninsured and who could not afford to pay for services. EMTALA does not have to do with provisions for nonsuburban clients, ensuring the poor use emergency services as primary care, or preventing free examinations to the poor

Acute respiratory acidosis can lead to ________, which affects neurological function and the cardiovascular system. A) hypercapnia B) carbon dioxide narcosis C) hypoventilation D) hyperventilation

A) In acute respiratory acidosis, increased carbon dioxide levels, also called hypercapnia, can affect neurological function and the cardiovascular system. Carbon dioxide narcosis occurs in chronic respiratory acidosis. Hypoventilation causes respiratory acidosis; it doesn't result from respiratory acidosis. Hyperventilation is related to respiratory alkalosis, not respiratory acidosis.

A client in the clinic asks the nurse about a medication seen on television. When the nurse suggests a generic, cheaper version of the medication, the client is resistant to this recommendation. This is an example of which of the following? A) Healthcare treatment choices that drive up costs B) Factually misleading advertisements by pharmaceutical manufacturers C) Client-focused care D) The need for stricter Food and Drug Administration (FDA) safeguards for consumers

A) Inappropriate healthcare treatment choices by consumers have also contributed to increases in healthcare costs. Mass advertising by pharmaceutical manufacturers and specialty treatment centers directed at consumers have contributed to these inappropriate treatment choices. However, this does not mean the advertisements are necessarily factually misleading. A client being guided in healthcare decisions by an advertisement is not an example of client-focused care or necessarily an argument for stricter FDA safeguards.

The nurse asks the client to repeat the information taught during the discharge teaching session. The client states, "I have forgotten everything you just said." Which action by the nurse would is appropriate at this time? A) Repeating the information and having the client write it down as the nurse teaches B) Having the client wait to ask questions until after the presentation C) Assigning another nurse to provide the teaching for the client D) Asking the client their preferred learning strategies

A) It is important for nurses to evaluate their own teaching and the content of the teaching program, just as they evaluate the effectiveness of nursing interventions for other nursing diagnoses. The nurse should not feel ineffective as a teacher if the client forgets some of what is taught. Forgetting is normal and should be anticipated. Having the client write down information, repeating it during teaching, giving handouts on the information, and having the client be active in the learning process all promote retention.

Why should job seekers in the healthcare sector pay attention to advances in healthcare technology? A) Advances in technology require specialized personnel. B) Advances in technology involve policies and strategies at the organizational level. C) Changing demographics increase the need for new jobs. D) Technology plays a role in health literacy.

A) Many advances in technology require specialized personnel, creating new opportunities for individuals seeking employment in the healthcare sector. These advances may involve politics and strategies at the organizational level, but that is not of primary interest to job seekers. Changing demographics do not necessarily involve technology. Technology does play a role in health literacy, but this is not of primary interest to job seekers

A nurse working on a medical-surgical unit has opted to return to school to earn a Bachelor of Science in Nursing (BSN) degree. After considering projected changes in healthcare and the population cared for in the community, which includes an expanding minority population composed largely of immigrants arriving from Central and South America as well as older adults as the fastest-growing demographic, the student might consider selecting which elective course? A) A course on medical Spanish B) A psychology course on young adults C) A personal finance class D) A class on the effect of illness on a young child

A) Minorities in the United States will likely be the majority by the year 2042. By becoming proficient at other languages, such as Spanish, the nurse will be better able to meet the needs of the clients who seek care within the community. The largest group of clients will be age 65 or older in the near future, so an extra class about an aging population would be more helpful than a class about children or young adults. The student might consider a class on the effects of finance and cost in the delivery of healthcare rather than personal finance.

An intervention that can be implemented independently by the nurse caring for a client with an acid-base balance is A) monitoring intake and output. B) drawing blood for ABGs. C) giving sodium bicarbonate infusions. D) administering oxygen via nasal cannula

A) Monitoring intake and output is an independent nursing intervention that does not require a provider's orders. Drawing blood for ABGs, giving sodium bicarbonate infusions, and administering oxygen via nasal cannula are all actions that can be performed by the nurse, but they must first be ordered by a provider.

The nurse is taking care of a client who is being discharged but will need home nursing care, physical therapy, and speech therapy. Which framework helps the client who has multiple care needs? A) Case management B) Client-focused care C) Managed care D) A health maintenance organization

A) Multidisciplinary teams led by a case manager are at the heart of successful case management. Case management is essential when a client has multiple care needs and requires the services of multiple providers. The goal of case management is to reach and then maintain the individual's optimum level of health, quality of life, and activities of daily living by ensuring that the individual's healthcare needs are met. Client-focused care is focused on the client's expressed needs but not specifically on providing the services of multiple providers, managed care provides high-quality care at a lower cost but is not specifically focused on providing the services of multiple providers, and HMOs are a kind of managed care.

As a working professional concerned with proper resource allocation, the nurse knows that one reason to join the American Nurses Association (ANA) is for which opportunity? A) To participate in national discussions about resources B) To draw attention to the nurse's credentials C) To determine how to fight for resources locally D) To advocate for resources through different business sources

A) Nurses must be aware of and participate in discussions that affect the allocation of healthcare resources in the workplace, in their communities, and at the federal level. Nurses are uniquely placed to advocate on behalf of clients when allocation of resources is being considered in their communities. They may advocate by talking with local legislators, writing to politicians, and engaging in discussions in their neighborhoods and social groups. As working professionals, nurses have the opportunity to participate in national discussions about resources through a number of professional organizations, such as the American Nurses Association. ANA membership as regards resource allocation is not a means of building the nurse's credentials, fighting for local resources, or enlisting business sources in the provision of resources.

A nurse is responding in the aftermath of a hurricane. Hundreds of clients demand attention. The nurse will implement which of the following in assessing the priority of these clients? A) Reverse triage B) Standard emergency department triage C) A disaster response plan D) American Nurses Association (ANA) ethics rules

A) Nurses perform triage every day in emergency departments. During a mass casualty event (more than 100 victims), the demand on nurses' knowledge and skills will be even greater. Mass casualty events call for the implementation of reverse triage, in which the most severely injured or ill victims who require the greatest resources are treated last to allow the greatest number of victims to receive medical attention. A disaster response plan involves the general response to an emergency. The American Nurses Association (ANA) guides nurses in making ethical decisions but does not specifically address the means of prioritizing client needs in a disaster.

The nurse in an urgent care center is assessing an adult client who is diagnosed with the flu. The nurse discusses the need for flu shots with the client, who states, "I cannot afford the shots. I do not have health insurance." Which suggestion by the nurse is most appropriate? A) Seek preventive care at the local health department. B) Find a primary care physician who will give free care. C) Obtain the flu shot at a local pharmacy. D) Get the shot every year in the emergency department.

A) Public health organizations, such as local health departments, are available for those who are uninsured or underinsured. This provides the client with health promotion and preventive measures as well as treatment when the client is ill. Using the emergency department for preventive care is part of the increased cost of healthcare. The nurse should not give the client a vague message that he or she needs need to find a care provider who gives free care. Rather, the nurse would refer the client to a specific place that can meet the client's needs. Flu shots at pharmacies require payment by the client and are not the solution for those with a low income and no insurance.

The nurse educator is teaching a group of nursing students about the methods that are used by insurance companies to ration healthcare resources. Which method does the educator include in the teaching session regarding this topic? A) Denying coverage for services not supported by research B) Covering clients for preexisting conditions only C) Providing health savings accounts for covered individuals D) Covering organ donations

A) Rationing is one means of allocating healthcare resources. Rationing is a method used by individuals, insurance companies, and the government to prevent increases in the cost of healthcare or to reduce the cost of healthcare. Methods used by insurance companies to ration healthcare resources include noncoverage (e.g., noncoverage of preexisting conditions for up to a year after enrollment) and denial of coverage for services (e.g., services deemed to be experimental or those that are not supported by scientific evidence that proves their efficacy). Covering clients for preexisting conditions only, providing health savings accounts, and organ donation are not methods of rationing used by insurance companies.

At a local seminar discussing healthcare resources, the nurse discusses the phenomenon that healthcare resources are declining while costs for healthcare are increasing. An older adult in attendance asks the speaker why it has become so difficult to obtain needed care and services. Which response by the nurse is the most appropriate? A) "There is increased cost due to the increased incidence of malpractice lawsuits." B) "There is a decrease in the number of adults needing care." C) "There are not enough medications produced for those who need them." D) "There are plenty of nurses but not enough doctors."

A) Services are reduced because of increasing costs and needs. One reason for increased costs is the practice of defensive medicine because of the risks of malpractice litigation. There is an increasing shortage of nurses and physicians. Over the next decade, there will be an increasing number of older adults needing medical care. The cost of supplies and medication, not the amount available, is the reason for decreased services.

An alkalinizing solution often given intravenously to clients with severe acute metabolic acidosis is A) sodium bicarbonate. B) sodium chloride. C) potassium chloride. D) dextrose.

A) Sodium bicarbonate is an alkalinizing solution often given intravenously to clients with severe acute metabolic acidosis. Sodium chloride and potassium chloride may worsen metabolic acidosis by increasing the chloride concentration. Dextrose may also worsen metabolic acidosis, especially in clients with type 1 diabetes, by increasing blood glucose levels and causing ketoacidosis.

The nurse is reviewing client records to determine which clients are good candidates for case management. Which client would the nurse choose as having the highest priority for assigning a case manager? A) A newborn who was diagnosed with cerebral palsy B) A pregnant woman with three other children C) A school-age child who was diagnosed with pneumonia D) An older adult who has been diagnosed with hypertension

A) Some clients are more likely to benefit from case management than others, including at-risk pregnant women, children with congenital conditions, and older adults with many chronic conditions. A newborn diagnosed with cerebral palsy has a congenital condition that will require lifelong care. The pregnant woman does not appear to be at risk from this description. The school-age child has an acute condition, not a chronic condition. The older adult has a chronic condition, but in the absence of other complicating factors, this one condition would not require case management.

A client asks the nurse, "How am I going to pay for a surgery? I'm broke, but I've been so sick." Which statement regarding the Affordable Care Act (ACA) may be applicable to this client's situation? A) "The ACA extends coverage to people who do not qualify for public assistance and whose employers do not offer health insurance." B) "Unfortunately, you will probably be denied coverage because of your existing condition." C) "Most insurance premiums will be more and some less; you will have to shop around." D) "Because the rate of inflation is slowing, the ACA will also help curb medical costs."

A) The ACA was created to ensure that all U.S. citizens have access to affordable, quality care and to curb the growth of healthcare costs. Under the ACA, a client cannot be denied coverage because of an existing condition. Insurance premiums should remain the same. The ability of the ACA to curb medical costs is irrelevant to this client's needs.

The nurse is working with an emergency response team following massive flooding caused by a hurricane. What will working with the Clinical Outreach Communication Activity (COCA) team enable the nurse to do? A) Have two-way communication with the Centers for Disease Control and Prevention (CDC) concerning infection risks. B) Facilitate communication between doctors in the field during a disaster and their healthcare team. C) Provide resources to the community during times of disaster. D) Provide expert advice to other nurses during natural disasters.

A) The CDC manages the COCA to ensure that clinicians have up-to-date information. The COCA is designed to provide two-way communication between clinicians and the CDC about emerging health threats such as pandemics, natural disasters, and terrorism. The COCA keeps a list of emergency preparedness and training resources offered by federal agencies and COCA partners. COCA does not enable communication between doctors and their teams in the field, provide resources to communities during disasters, or enable communication between nurses during disasters.

The nurse is caring for a client who received analgesic medication via central line to treat pain associated with cancer. After reassessing the client's response, which section of the PIE record will the nurse use when documenting the client's care? A) Evaluation B) Progress notes C) Problem D) Intervention

A) The PIE documentation model groups information into three categories: problems (P), interventions (I), and evaluation (E) of nursing care. Reassessing the client's pain level after medication administration is considered evaluation and would be documented under "E." Interventions such as medication administration would labeled "I," whereas the problem statement would be labeled "P." Progress notes are not part of the identified labels of PIE charting.

A home health nurse is admitting a new client to the agency who was recently discharged from the hospital with a new diagnosis of pulmonary fibrosis. What is the best way for the nurse to evaluate whether the client is able to set up and administer a nebulizer treatment? A) Direct observation of behavior B) Written description by the client of the treatment C) Oral description by the client of the treatment D) The client reports success or failure with the treatment at a follow-up appointment

A) The best way for the nurse to evaluate whether this client is able to set up and administer a nebulizer treatment is by direct observation of the client doing it. A written or oral description of the treatment does not as directly demonstrate that the client can set it up and administer it, and the client reporting success or failure at a follow-up visit doesn't give the nurse a direct means of evaluating what the client understands.

The nurse has completed discharge teaching for a client with an anxiety disorder. Which client statement indicates that client teaching about respiratory alkalosis has been effective? A) "I will see my counselor on a regular basis." B) "I will breathe faster when I am feeling anxious." C) "I will eat more bananas at breakfast." D) "I will not take antacids when I have heartburn."

A) The client understands that reducing anxiety can reduce hyperventilation and respiratory alkalosis. Seeing a counselor can help the client develop alternative strategies for dealing with anxiety. Eating bananas is more appropriate for the client at risk for metabolic alkalosis who is on diuretics. Breathing faster will increase hyperventilation. Taking too many antacids is associated with metabolic alkalosis.

A client who had outpatient surgery is given an instruction sheet in preparation for discharge. When the nurse asks if the instructions are clear, the client says, "I'll read them later when I have my glasses; besides, you told me everything I need to know." Based on these statements, what would the nurse suspect? A) The client may be unable to read the instructions. B) The client already knows the information. C) The client does not want the written information. D) The client is ready to learn.

A) The client who refuses to read instructions may not be able to read. The nurse should assess the client's ability to read to ensure proper treatment and to evaluate the client's understanding of the information. It is unlikely that the client does not want the information. The client said that he'd read the instructions later, and so is not opposed to written information. Although the nurse did provide verbal instruction, often clients forget portions because of the amount of information presented at discharge. The client may be ready to learn, but the client's inability to read may be masking that fact.

The nurse is planning care for a client who has been admitted to the unit with a salicylate overdose. When preparing the plan of care, the nurse considers which to be a priority nursing diagnosis? A) Ineffective Breathing Pattern B) Powerlessness C) Risk for Injury D) Impaired Mobility

A) The client with a salicylate overdose is at risk for hyperventilation, which can lead to respiratory alkalosis. There is not enough information to know whether the client's mobility is impaired. Risk for Injury and Powerlessness are diagnoses to be considered for this client, but the highest priority is respiratory function.

A client with hyperaldosteronism is admitted to the unit and is at risk for impaired gas exchange. In which position should this client be placed to enhance gas exchange? A) Fowler position B) Prone position C) Left side-lying position D) Right Sims position

A) The client with hyperaldosteronism with metabolic alkalosis will likely have reduced oxygenation. The Fowler position will facilitate alveolar ventilation with improved oxygenation. Side-lying and prone positions do not facilitate needed lung expansion.

The nurse is caring for a client admitted with renal failure and metabolic acidosis. Which clinical manifestation would indicate to the nurse that planned interventions to relieve the metabolic acidosis have been effective? A) Decreased respiratory depth B) Palpitations C) Increased deep tendon reflexes D) Respiratory rate of 38

A) The client with metabolic acidosis will have an increased respiratory rate and depth, called Kussmaul respirations. Signs that care has been effective would include a decrease in the rate and depth of respirations. An increased respiratory rate, as indicated by a respiratory rate of 38, would indicate continued metabolic acidosis. Increased deep tendon reflexes and palpitations are not associated with metabolic acidosis.

The nurse is providing care to a client recently extubated for treatment of aspiration pneumonia and respiratory acidosis. Which action by the nurse provides an optimum environment for this client? A) Allowing family members to remain with client as much as possible B) Restraining the client C) Placing the client in a side-lying position D) Administering narcotics for pain

A) The client with respiratory acidosis often experiences anxiety. This client would benefit from having a family member in the room to provide reassurance. Restraining the client will increase levels of agitation. The client with respiratory failure would benefit most from the semi-Fowler or Fowler position to increase ventilation. Narcotics will depress the respirations and increase respiratory acidosis. A nonnarcotic pain reliever would be considered if this client were experiencing pain.

The nurse is analyzing the client's arterial blood gas report, which reveals a pH of 6.58. The client has just suffered a cardiac arrest. Which consequences of this pH value does the nurse consider for this client? A) Decreased cardiac output B) Increase magnesium levels C) Decreased free calcium in the ECT D) Increased myocardial contractility

A) The nurse knows that severe acidosis (pH of 7.0 or less) depresses myocardial contractility, which leads to decreased cardiac output. Acid-base imbalances also affect electrolyte balance. In acidosis, calcium is released from its bonds with plasma proteins, increasing the amount of ionized (free) calcium in the blood. Magnesium levels may fall in acidosis.

The nurse is caring for a client who has recently received a permanent colostomy. The client will be going home in several days and requires discharge teaching. What should the nurse do when organizing the teaching experience? A) Ask the client to tell the nurse what he knows about caring for the colostomy. B) Make sure the client's spouse is present before the teaching session begins. C) Start from the beginning and proceed through all steps required to perform colostomy care. D) Break the information into small sessions to enhance learning.

A) The nurse should find out what the client knows, and then proceed to the unknown. This gives the client confidence. This information can be elicited either by asking questions or by having the client take a pretest or fill out a form. Going over information already learned is not practicing good time management for the nurse or the client. Unless the client has attention problems or is an older adult, breaking up the sessions is probably unnecessary. Having the spouse present is always a good idea, but finding out what the client already knows is more important when organizing teaching.

The nurse is caring for a preschool-age client who suffered brain damage following a car accident. The client has a tracheostomy, is ventilator-dependent, and will be discharged from the hospital into the family's care. The family wants to care for the child at home but does not have the resources for 24-hour care. Which action by the care manager is appropriate? A) Making referrals to a variety of community-based agencies that can meet the family's needs B) Telling the family that it is impossible to provide care at home C) Arranging for the child to be sent to a long-term healthcare facility D) Contacting local nurses in the community to provide the assistance that the family needs

A) The referral process is a systematic problem-solving approach that helps clients use resources that meet their healthcare needs. The nurse should not tell the family they cannot care for the client at home, nor should the nurse arrange for the child to be sent to a long-term care facility, because these actions do not reflect the family's wishes. Contacting local nurses would not be an efficient way to manage this client's care.

The nurse is assigned to provide care to a client with chronic obstructive pulmonary disease (COPD). Overnight, the client's oxygen saturation levels decreased and the client has been placed on oxygen by the respiratory therapist. To review specific information about the care received from the respiratory therapist, which portion of the medical record should the nurse review? A) The consultation report B) The nurses' notes C) The medication record D) The diagnostic report

A) The traditional client record is a source-oriented record, in which each individual or department makes notations in a separate section or sections of the client's chart. The consultation report section of a source-oriented record includes various reports about the client from physical therapy, respiratory therapy, and occupational therapy. The nurses' notes section is specific to nursing care and includes pertinent client information; the medication record section has the dosage, route, time, and date of regularly administered medications; and the diagnostic report section has various laboratory, x-ray, and scan reports on the client.

A railway accident causes the release of a dangerous chemical compound into the atmosphere. The nurse providing rapid triage and emergency treatment for clients in an effort to stabilize them knows that which is the primary purpose of the warm zone in this incident? A) Decontamination B) Rapid triage C) Reverse triage D) Emergency medical treatment

A) The warm zone (also referred to as the yellow, contamination, or contamination reduction zone) is located at least 300 feet from the outer edge of the hot zone. Although the primary purpose is decontamination, rapid triage and emergency treatment to stabilize victims may also take place in the warm zone. Individuals who have the highest levels of contamination are treated with the highest priority. Personal protective equipment (PPE) is required in this zone.

The nurse educator is presenting information to a group of nursing students regarding uninsured and underinsured clients. Which of the following is the best example of this problem for the educator to share with the students? A) "Delays of diagnoses lead to higher mortality and morbidity rates." B) "Delays in health coverage for children put the health provider at risk for litigation." C) "Immunizations are free for children at public health clinics." D) "Older adults are less likely to be treated for falls."

A) Those who are not insured, or are underinsured, often do not seek treatment in a timely manner due to finances. As a result, diagnosis is made in the later stages of the disease, resulting in decreased chance of survival and an increased cost of treatment. The exposure of healthcare providers to litigation is not an aspect of this issue. Children with healthcare coverage receive preventive care such as immunizations and are more likely to stay healthy and do well in school. Adults age 65 and older are eligible for Medicare and have access to healthcare for falls and other medical problems.

The nurse is caring for an adult client who has been diagnosed with high cholesterol. Which is important for the nurse to consider when teaching this adult client? A) Adults are more oriented to learning when the material is useful immediately. B) Adults are more likely to adhere to a regimen than are children. C) Adults usually can find information on their own. D) Adults do not need to be evaluated for understanding as children do.

A) When teaching a client, the nurse considers that most people learn and retain information if the information is immediately useful. Some clients can find information on their own; however, not all information that the client can find is factual, and clients should be taught how to discern the difference between trustworthy information and unreliable and potentially dangerous information. All clients need to be evaluated to ensure that the right information was retained. Adults will not necessarily adhere to a regimen more than children will. Effective teaching and the client's readiness to learn help with adherence.

A nurse on the pediatric unit contemplates the changes in healthcare insurance for 2014 when speaking with other colleagues. Which aspects of the Affordable Care Act (ACA) affect children favorably? Select all that apply. A) Insurance companies cannot deny coverage based on preexisting conditions. B) Insurance companies cannot drop children who have serious illnesses. C) Benefits are paid through private benefactors. D) All children are covered regardless of whether the parents are covered. E) The government must make appropriate arrangements for children based on need.

A, B) Children with healthcare coverage receive preventive care and are more likely to attend school regularly and have better focus. The ACA will affect children favorably because insurance companies will no longer be able to deny coverage to children based on preexisting conditions or drop insured individuals and/or their dependents who experience serious illness. Under the ACA, benefits are not paid through private benefactors. Coverage is intended for all U.S. citizens, not just children and not only based on an assessment of need.

A nurse is planning a community health fair at a local community center. Which goals regarding health promotion does the nurse plan to highlight at the event? Select all that apply. A) The ability to change and modify goals as health needs change B) The ability for clients to be able to assess and evaluate their health needs C) The ability for the client to promote health in other individuals D) The ability to promote cost-saving techniques to healthcare providers E) The ability to prevent disease by imitating nursing techniques

A, B) The nurse has an integral role in health promotion. The nurse's aim should be to teach clients how to remain healthy, thus preserving wellness. The overarching goal is to ensure that clients understand the importance of setting health goals for themselves and their children, and that clients are able to assess, implement, evaluate and, as their health needs change, modify them. This does not include teaching clients to promote health in others, saving costs to healthcare providers, or actively preventing disease by imitating nursing techniques.

A nurse has just been hired as a medical information system (MIS) trainer at a hospital where an electronic medical record system is being installed. The nurse has been asked to assess the security of clients' medical records. According to HIPAA's Security Rule, which recommendations by the nurse will enhance security? Select all that apply. A) Assign each staff member a unique username and password. B) Install a firewall. C) Store computer-generated worksheets in a locked vault. D) Turn monitors away from view when unattended. E) Assign each unit unique passwords.

A, B) To comply with HIPAA's Security Rule, institutions should assign individual passwords to each staff member for logging on and off computer files. Firewalls should be installed to enhance the security of client records. Client information should not be displayed on unattended terminals, regardless of which direction the monitor faces, and computer-generated worksheets should be shredded when no longer needed.

The local clinic staff has noticed an increase in the flu this season and requested more vaccine from the Centers for Disease Control and Prevention (CDC). The nurse knows that the CDC will control the vaccine distribution by considering which factors? Select all that apply. A) The amount of vaccine being requested by local health departments B) The speed at which the vaccine becomes available C) Who gets the available vaccine D) Ensuring that only medical personnel can receive the vaccine E) Ensuring that only babies and military personnel receive the vaccine

A, B, C) During 2009, there was an outbreak of H1N1 influenza. Initially an inadequate supply of vaccine was available, but as H1N1 began to spread, the CDC realized that guidelines had to be developed and implemented to ensure that the most vulnerable individuals had first access to the vaccine. As more vaccine became available, the CDC controlled its distribution by considering factors such as the amount of vaccine being requested by local health departments, the speed at which the vaccine was becoming available, and the need to ensure availability for not only the community but also active-duty military personnel. The CDC will not act to ensure that only medical personnel, babies, or military personnel receive the vaccine.

The nurse is evaluating care provided to a client with respiratory alkalosis. Which outcomes indicate that nursing care has been effective for this client? Select all that apply. A) Respiratory rate 18 and regular B) Sleeping through the night C) Gait steady D) Consistent body weight E) Using prescribed bronchodilators

A, B, C, D) Appropriate outcomes for the care of a client with respiratory alkalosis include normal respiratory rate and rhythm, no episodes of injuries, and maintenance of fluid balance. Ability to sleep through the night would indicate a reduction in anxiety, which is a risk factor for the development of respiratory alkalosis. Bronchodilators are not used to treat this acid-base imbalance.

The nurse is preparing discharge instructions for an older adult client recovering from respiratory acidosis caused by restrictive lung disease and pneumonia. Which topics should the nurse include in the discharge teaching for this client? Select all that apply. A) Obtain annual influenza immunization. B) Engage in frequent hand washing. C) Avoid crowds. D) Cover the nose and mouth when coughing. E) Restrict fluids.

A, B, C, D) For the client with a history of chronic lung disease and pneumonia, the nurse should instruct on the importance of receiving annual influenza immunizations, frequent hand washing, avoiding crowds, and covering the nose and mouth when coughing. Fluids should be encouraged to ensure that respiratory secretions are thin.

A nurse is providing a series of educational workshops for caregivers of older clients interested in promoting the health and well-being of their clients. Which would be appropriate topics for this group? Select all that apply. A) Fall prevention B) Medication use and side effects C) Safe driving evaluations D) Advance directives E) Responsible sexual behavior

A, B, C, D) It is important for caregivers of older clients to learn about how to prevent falls, medication use and side effects, safe driving evaluations, and advance directives, but it would be much less appropriate for this age group to include teaching about responsible sexual behavior, which would be better addressed to younger adult clients.

When the nurse receives a telephone order from the healthcare provider's office, which guidelines should the nurse use to ensure the order is correct? Select all that apply. A) Ask the provider to repeat or spell out medication. B) Read the order back to the provider. C) Ask the provider to speak slowly. D) Know agency policy for telephone orders. E) Sign the provider's name and credentials.

A, B, C, D) When receiving a telephone order from a provider, the nurse should ask the provider to repeat or spell out any medications and speak slowly. The nurse should also read the order back to the provider once the order is complete. In addition, the nurse should know the agency's policy regarding telephone orders. The nurse should not sign the provider's name and credentials; the nurse only transcribed the prescription, so the provider will need to countersign it within a time period prescribed by organizational policy.

The nurse identifies the diagnosis Risk for Impaired Gas Exchange to guide the care of a client with metabolic alkalosis. Which assessment data supports this nursing diagnosis? Select all that apply. A) Respiratory rate 8 per minute B) Oxygen saturation 89% C) Urine output 25 mL/hr D) Restlessness and agitation E) Weight loss of 3 kg overnight

A, B, D) Respiratory compensation for metabolic alkalosis depresses the respiratory rate and reduces the depth of breathing to promote carbon dioxide retention. The depressed respiratory drive associated with metabolic alkalosis can lead to hypoxemia and impaired oxygenation of tissues. Oxygen saturation levels of less than 90% indicate significant oxygenation problems. Changes in mental status or behavior may be early signs of hypoxia. Urine output less than 30 mL/hr would indicate fluid volume deficit. Weight is used as an indicator of fluid balance. A rapid weight change would indicate fluid volume deficit.

A client states to the nurse, "I experience shortness of breath and dizziness every time I get into an elevator." Which actions by the nurse are appropriate based on this data? Select all that apply. A) Assist the client to rethink the degree of anxiety associated with elevators. B) Ask the client how he has survived in life so far with elevators. C) Instruct the client in deep breathing exercises. D) Suggest that the client should avoid elevators. E) Tell the client that elevators are completely safe.

A, C) Deep breathing exercises can help the client reduce the anxiety associated with a fearful situation, such as entering an elevator. Assisting the client to rethink the degree of anxiety associated with elevators helps the client learn to manage the anxiety. Suggesting that the client avoid elevators will not help the client. Asking the client how he has survived in life so far with elevators will not help the situation. Telling the client that elevators are completely safe might not be true and should not be said to the client.

The nurse is caring for the client experiencing hypovolemic shock and metabolic acidosis. Which therapies would the nurse question if planned for this client? Select all that apply. A) Monitor weight on admission and discharge. B) Monitor ECG for conduction problems. C) Limit the intake of fluids. D) Administer sodium bicarbonate. E) Keep the bed in the locked and low position.

A, C) The treatment for hypovolemic shock would include the administration of fluids, not limiting fluids. Patients being treated for hypovolemia and metabolic acidosis will require daily weights, not a weight on admission and then discharge. Administering sodium bicarbonate and monitoring ECGs are appropriate for the client with metabolic acidosis. The client recovering from hypovolemic shock and metabolic acidosis is at risk for injury, so the bed should be kept in the locked and low position.

The nurse is preparing to document care provided to a client during the day shift. The nurse notes that the client experienced an increased pain level while ambulating and thus required an extra dose of pain medication; took a shower; visited with family; and ate a small lunch. Which information is important to include during the oral end-of-shift reporting? Select all that apply. A) The extra dose of pain medication B) The client's visit with family C) The client's response to ambulation D) The last antibiotics given E) The client's taking a shower

A, C) To best provide for the client's safety, the nurse should pass on information about the client's response to ambulation so that the oncoming staff can take fall precautions. The nurse should also report any as-needed medications that were given and when they were last administered. The client's visit with family need not be mentioned at change of shift but should be documented. Likewise, taking a shower does not need to be reported, only documented. Antibiotic administration would be reflected on the medication administration record (MAR).

A hospital in the community has been notified of a multi-car crash on the interstate that will result in the transfer of many injured clients to the hospital. As part of the emergency response, the charge nurses in the emergency department (ED) and intensive care unit (ICU) are responsible for which tasks? Select all that apply. A) Assigning care for the clients as they are admitted to the unit B) Exceeding their scope of practice when and if required C) Assessing the priority of the current clients for the ED or ICU D) Delegating staff nurses to gather needed supplies for the arriving clients E) Providing any care that any patient needs

A, C, D) Nurses must observe both the physical and mental status of victims and ensure appropriate triage and treatment. They should not use their time to provide care that will be of minimal or questionable benefit. During the crisis, nurses should be constantly aware of their defined scope of practice and must not exceed it even when circumstances would seem to dictate otherwise. The charge nurses in each unit are responsible for assessing the current clients and recommending to the physicians those clients who can be moved to make room for clients needing more immediate care. The charge nurse would delegate the gathering of supplies to the staff nurses. The charge nurse would also assign care for the clients who are admitted to the unit.

The client with an anxiety disorder is ready to be discharged from the unit. What should the nurse plan to teach this client and family in preparation for discharge? Select all that apply. A) Refer the client for counseling. B) Instruct the client to eat foods high in acid. C) Teach the client the signs of impending panic attack. D) Advise the client to breathe into a paper bag when feeling anxious. E) Instruct the client to breathe slowly.

A, C, E) Teaching the client to breathe slowly helps the client manage hyperventilation at home. The client with an anxiety disorder should be referred to counseling to assist with management of the disorder and should be taught signs of an impending panic attack. Eating foods high in acid will not counteract the results of hyperventilation. The use of paper bags has been a recommended treatment for hyperventilation; however, it can also cause hypoxia.

The nurse is conducting a health history on a client who is being admitted to a medical-surgical unit for the treatment of chronic pain. The client is concerned about privacy and asks why it is necessary for the nurse to ask for private information and then document it in the medical record. Which response by the nurse is most appropriate? A) "You will be able to read the record and review your care." B) "Documentation decreases the likelihood that you will have to repeat this information to others who will care for you." C) "Your family can review the record and ensure that your care is appropriate." D) "A record ensures there are no breaches of confidentiality."

B) A client's record serves as a vehicle by which different health professionals who interact with the client communicate with one another. This prevents fragmentation, repetition, and delays in client care, and it relieves the client from having to repeat information to each provider offering care. The client can read the record, but that is not a reason to keep one. The client's family does not have access to the record. Recordkeeping does not prevent breaches of confidentiality.

The nurse performing a blood draw for arterial blood gases first performs a modified Allen test for what purpose? A) To reduce the risk of bleeding or bruising of the arm B) To determine if arterial puncture can safely be performed C) To determine the oxygen saturation of the blood in the artery D) To determine the pressure of the blood in the artery

B) A modified Allen test is a measure of ulnar patency. The patient elevates the hand and repeatedly makes a fist while the examiner places digital occlusive pressure over the radial and ulnar arteries of the wrist. The hand will lose its normal color. Digital pressure is released from one artery while the other remains compressed. The return of color indicates that the hand has good collateral supply of blood and that arterial puncture can safety be performed. The modified Allen test does not measure oxygen saturation or artery pressure, and it does not reduce the risk of bleeding or bruising.

A client with metabolic alkalosis is experiencing numbness around the mouth and tingling of the fingers. What should the nurse explain as the reason for these manifestations? A) "Because you are breathing so fast, the oxygen is not getting to your nerve endings." B) "Your health problem affects calcium in your body, which causes the tingling around your mouth and fingers." C) "You have a buildup of carbon dioxide in your blood." D) "You don't have enough potassium in your body, so the tingling around your mouth and fingers will occur."

B) Alkalosis increases binding of extracellular calcium to albumin, reducing ionized calcium levels. As a result, neuromuscular excitability increases, and manifestations similar to hypocalcemia develop. These manifestations include circumoral and distal extremity paresthesias. Rapid breathing is not reducing the amount of oxygen reaching the nerve endings. Excessive carbon dioxide would lead to acidosis. Respiratory alkalosis is not caused by an imbalance of serum potassium.

The nurse educator is preparing to teach a group of nursing students how to navigate the internet to research healthcare information. Which does the educator plan to include during lecture? A) A directory of campus internet sites of interest B) How to search for and evaluate health information C) A directory of libraries D) Information technology instruction

B) Campus health centers that use the Internet as a tool for health education must train nursing students regarding how to search for and evaluate the health information they find. Sites of interest for the campus would not directly impact the nursing program. Information technology is a subject that teaches nurses how to use technology for the delivery of care and communication. Libraries are important, but knowing about them would not be a part of this presentation.

The nurse is documenting care in a client's medical record. The nurse provides narrative documentation only for abnormal assessment findings. Based on this information, which type of charting is the nurse using? A) Computerized documentation B) Charting by exception (CBE) C) SOAP charting D) Focus charting

B) Charting by exception (CBE) is a documentation system in which only abnormal or significant findings or exceptions to norms are recorded. Flow sheets, standards of nursing care, and bedside access to chart forms are all incorporated into CBE. Computerized documentation is a way to manage the volume of information required in a client's chart, and different systems may include a variety of setups and programs. Focus charting is organized into data, action, and response sections, referred to as DAR. SOAP charting is a way to organize data and information in the client's record: S indicates subjective data, O indicates objective data, A indicates assessment, and P indicates plan of care.

A new nurse on a unit asks to speak to the nurse manager because several clients have complained that family members were able to hear the verbal report outside their loved one's room during nursing rounds. The nurse manager asks the nurse for suggestions that could enhance client privacy. Which suggestion by the new nurse is appropriate? A) Nursing rounds should take place in each client's room. B) The unit should be closed to family and visitors during rounds. C) Nurses should tape-record their reports outside the room. D) Clients should be allowed to choose whether a written or oral report is used.

B) Closing the unit during nursing rounds permits nurses to talk freely and relate important information regarding clients and their care while minimizing the risk of violating client privacy. Rounds could take place in each client's room, but if a client's family is present, confidentiality is compromised. The nursing staff and manager should determine what form of report is used, keeping in mind the confidentiality of the client. Taped reports are acceptable if conducted in private but not appropriate if conducted out on the unit.

What nursing concept is essential to ensure continuity of client care when a nurse is managing the care of a client? A) Ethics B) Communication C) Advocacy D) Teaching and Learning

B) Communication among members of the collaborative team of healthcare providers is essential to ensure continuity of client care. Although ethics, advocacy, and teaching and learning are important for managing care in general, they do not directly influence the continuity of care.

The nurse provides medication teaching for a client who will be going home on new medications. Which statement by the client best illustrates compliance with the medication plan? A) "I think you should have waited until I was ready to go home. Maybe I'd remember better." B) "I'm glad to know about my new medications. It makes taking them all a lot easier." C) "If I take my medications as prescribed, I'll feel better." D) "I already knew most of what you told me."

B) Compliance is best illustrated when the individual recognizes and accepts the need to learn, then follows through with appropriate behaviors that reflect learning. Learning about the medications helps the client understand why they are prescribed and improves the possibility for following the prescribed regimen. Statements of prior knowledge do not necessarily lead to compliance, and neither does merely restating the advice of the healthcare provider.

A group of nurses attend an in-service regarding emergency preparedness for the hospital. One of the nurses has three small children and lives in a two-story house in the suburbs. After the class, the nurse plans to initiate which action to enhance family safety? A) Training her family in performing nursing interventions to take part in an emergency response B) Obtaining a fire escape ladder for the second floor of the home C) Developing a plan for her family to join her in the event of an emergency D) Ensuring she and her family move to a safe area unlikely to be involved in a disaster

B) During the preparedness phase, nurses must also develop an emergency plan for themselves and their immediate families. When this plan is complete, nurses can be confident that their families are prepared to weather an emergency in relative safety. With this assurance, nurses who choose to assist in a disaster or who are required to remain at the hospital during a disaster will be able to leave their families without delay and remain available until no longer needed. Part of this plan should include means of escape such as fire escape ladders. The nurse's family is not qualified to perform nursing interventions and should go to a safe place in an emergency, not join the nurse where she is. No place is completely safe from all potential disasters.

A common cause of metabolic acidosis is A) hyperventilation in a client with anxiety. B) high blood glucose in a client with type 1 diabetes. C) vomiting in a client with a gastrointestinal infection. D) opiate overdose in a client with depression.

B) High blood glucose that leads to diabetic ketoacidosis is a common cause of metabolic acidosis. Vomiting can lead to metabolic alkalosis, hyperventilation can lead to respiratory alkalosis, and opiate overdose can lead to respiratory acidosis.

The nurse assumes care for a client who was brought to the hospital after a morphine overdose. What acid-base imbalance does the nurse expect to observe in this client? A) Respiratory alkalosis B) Respiratory acidosis C) Metabolic alkalosis D) Metabolic acidosis

B) Morphine is a narcotic and generally acts to decrease or suppress respirations; therefore, this client is probably hypoventilating. The expected acid-base imbalance would be respiratory acidosis. Respiratory alkalosis, metabolic acidosis, and metabolic alkalosis are caused by many conditions, none of which are related to this client's morphine overdose.

A novice nurse asks the preceptor why the staff spends time talking about clients between shifts when the oncoming nurses can read the clients' charts instead. Which is the best response by the preceptor? A) "Maybe we should suggest primary nursing as an alternative." B) "Change-of-shift reporting ensures that oncoming staff know the most critical information about the clients they'll be caring for." C) "Shift changes have always been done this way." D) "You're right. Talking about clients during shift changes is a waste of time."

B) Nurses often do not have time to read clients' charts prior to assuming care, which could result in errors and assumptions. By participating in change-of-shift reports, outgoing nurses can ensure that oncoming staff are aware of critical information. The preceptor should not tell the new nurse that change-of-shift reports are a waste of time because these reports allow for communication of valuable client data. Stating that shift changes have always been done a certain way does not help the novice nurse understand why a change-of-shift report is necessary. Primary nursing promotes continuity of care, but even the primary nurse would need to be informed of client changes that occurred during his or her absence.

A client is brought to the emergency department (ED) with rapid breathing after learning of a family member being killed in a house fire. What should the nurse do first to help this client? A) Coach to slow the breathing. B) Move to a quiet, calm environment. C) Provide a sedative. D) Ask for a psychiatric consultation.

B) Nursing care is focused on reducing anxiety through manipulation of the environment to reduce stimuli and to create a sense of peace. This restful environment will help the client breathe more slowly and effectively. Once the environment is controlled, the nurse can begin to implement interventions to help the client slow the breathing rate. A sedative may be prescribed; however, this would not be the first intervention. A psychiatric consult might be indicated for someone with a history of anxiety or panic attacks that lead to the development of respiratory alkalosis. Because this client has had a shock, a psychiatric consultation would not be indicated at this time.

The nurse is caring for a client with metabolic acidosis. Which goals are appropriate for this client? Select all that apply. A) The client will maintain a respiratory rate of 30 or more. B) The client will describe preventative measure for the underlying chronic illness. C) The client will maintain baseline cardiac rhythm. D) The client will remain in a pH range from 7.25 to 7.35. E) The client will take potassium supplements to increase potassium levels.

B) Planning for the client with metabolic acidosis involves identification and treatment of the underlying cause and restoration and maintenance of acid-base balance. The client should be able to describe preventative measures for the underlying chronic illness that caused the metabolic acidosis to occur and maintain the baseline cardiac rhythm. The pH should be maintained between 7.35 and 7.45. The client's respiratory rate should be within normal range for age and condition. Taking a potassium supplement may cause hyperkalemia, which decreases cardiac output and worsens metabolic acidosis.

The nurse is assessing an older adult client in the free clinic. The nurse notes that the client's cholesterol level is higher than the target and has increased since the client's last visit. The nurse assesses the client for possible causes for this increase. After determining that the client has not changed dietary and exercise habits, which conclusion by the nurse is the most appropriate? A) The client has increased egg consumption. B) The client may not be taking medications as prescribed. C) The client needs more aerobic activity. D) The client may be experiencing high triglyceride levels.

B) Rationing is a method used by individuals, insurance companies, and the government to prevent increases in the cost of healthcare or to reduce the cost of healthcare. Individuals ration when they decide to provide self-care for an illness or injury rather than seeking care from a healthcare provider. In the case of this client, rationing of prescribed medications is likely the issue because dietary and exercise habits have not changed, which rules out increased egg consumption or the need for more aerobic activity. Concluding that the client may be experiencing high triglyceride levels does not identify a reason for the increased cholesterol level.

A client with metabolic acidosis has been admitted to the unit from the emergency department (ED). The client is experiencing confusion and weakness. Which nursing intervention is the priority for this client? A) Placing the client in a high-Fowler position B) Protecting the client from injury C) Administering sodium bicarbonate D) Providing the client with appropriate skin care

B) The client with metabolic acidosis may have symptoms of drowsiness, lethargy, confusion, and weakness. A priority of care would be preventing injury to the client. Medication administration is a physician order. Skin care would not be a priority on admission. The high-Fowler position would not be the safest position for the confused client.

The nurse suspects a client with one functioning lung is developing chronic respiratory acidosis. Which manifestation did the nurse most likely assess in this client? A) Warm, flushed skin B) Daytime sleepiness C) Irritability D) Blurred vision

B) The manifestations of acute and chronic respiratory acidosis differ. The client with chronic respiratory acidosis will demonstrate daytime sleepiness. The client with acute respiratory acidosis may demonstrate warm, flushed skin, irritability, and blurred vision from the acute decline in oxygenation.

A client with a suspected acid-base imbalance has arterial blood gases tested. The test reveals a serum bicarbonate level of 22 mEq/L. The nurse understands that this bicarbonate level is A) slightly high. B) slightly low. C) extremely high. D) within normal range.

B) The normal serum bicarbonate level is 24-28 mEq/L. Therefore, the nurse would understand that the client's bicarbonate level is slightly low.

The nurse is caring for a client with a new tracheostomy. After completing a teaching session on tracheostomy care, what should the nurse include in the documentation? A) The language used for teaching B) The need for additional teaching C) The client's questions after the teaching session D) The supplies required for teaching

B) The parts of the teaching process that should be documented in the client's chart include the need for additional teaching. Documenting the client's language is not necessary as it should already be in the nursing history. Supplies required for teaching are not documented. The client's questions are not documented, but the client's understanding at the end of the session is documented.

A client with severe metabolic alkalosis is admitted to the unit. Which is the priority for the client? A) Administering medication for metabolic alkalosis B) Monitoring oxygen saturation C) Teaching the client the risk factors for metabolic alkalosis D) Setting goals for the client with metabolic alkalosis

B) The priority for this client is monitoring oxygen saturation. The depressed respiratory drive that often accompanies metabolic alkalosis can lead to hypoxemia and impaired oxygenation of the tissues. Administering medications will be needed as a treatment, but the priority is to discover the cause. Teaching the client and setting goals are important aspects of nursing care but are not the priority.

A client is admitted with manifestations of metabolic alkalosis. Which diagnostic test findings support the admitting diagnosis? Select all that apply. A) Serum glucose level 142 mg/dL B) Blood pH 7.47 and bicarbonate 34 mEq/L C) Intravenous pyelogram shows kidney stones D) Bilateral lower lobe infiltrates noted on chest x-ray E) Electrocardiogram changes consistent with hypokalemia

B, E) In metabolic alkalosis, the blood pH will be greater than 7.45 and the bicarbonate level greater than 26 mEq/L. The ECG pattern shows changes similar to those seen with hypokalemia. Serum glucose levels, kidney stones, and lower lobe infiltrates are not associated with metabolic alkalosis.

A nurse is performing START triage for clients injured in a terrorist attack. Which client would the nurse classify as expectant? A) Client is breathing but has an absent radial pulse. B) Client has a respiratory rate below 30. C) Client is apneic after positioning of an airway. D) Client is breathing adequately with a radial pulse but does not obey commands.

C) A client who is apneic after the positioning of an airway would be tagged as black, expectant, meaning the client is not expected to survive. The other clients would all be classified as requiring immediate intervention.

The nurse is providing care to an older adult client diagnosed with respiratory alkalosis. The nurse states to the client, "Look into my eyes and breathe with me so that we can slow down your breathing rate." The client continues to look down and refuses to make eye contact with the nurse. The client's daughter later asks you to teach her how to help her mother to control her breathing. When documenting this client's care, which statement is appropriate for the nurse to include? A) "The client is noncompliant with suggested treatment plan." B) "The client is unable to understand and follow directions." C) "The client did not feel comfortable making eye contact during nursing care." D) "The client's daughter may be abusive."

C) A method that is often used to control breathing for client's experiencing hyperventilation is eye contract with the nurse during breathing exercises. However, some clients may feel uncomfortable making eye contact for personal reasons. Documenting this finding in the medical record is appropriate. Saying the client is noncompliant is nontherapeutic and labels the client. There is no indication that the client does not understand the instructions. The client's daughter wishes to help her mother control her breathing. This is not indicative of abuse.

A client is admitted to the hospital with sudden, severe abdominal pain. The client is diagnosed with respiratory alkalosis. Which arterial blood gas value does the nurse document to support this diagnosis? A) pH is 7.33 and PaCO2 is 36. B) pH is 7.51 and HCO3 is 30. C) pH is 7.47 and PaCO2 is 25. D) pH is 7.35 and PaO2 is 88.

C) Acute pain usually causes hyperventilation, which causes the PaCO2 to drop and the client to experience respiratory alkalosis. The pH would denote alkalosis and would be higher than 7.45. HCO3 would trend downward as the kidneys begin to compensate for the alkalosis by excreting HCO3. The PaO2 is likely to be normal unless the client has been hyperventilating for a long time and is beginning to tire.

During a home visit, the nurse evaluates care provided to a client with type 1 diabetes mellitus and a history of metabolic acidosis. Which outcome indicates that the care of this client has been successful? A) The client is injecting insulin into thigh muscle. B) The client is taking laxatives three times a week to ensure adequate bowel movements. C) The client is eating three balanced meals per day with two snacks. D) The client is taking aspirin 325 mg every 6 hours to treat arthritis pain.

C) Adequate nutrition is necessary to prevent the buildup of acids in the blood. Incorrect administration of medication could cause a metabolic problem in the client with diabetes. The use of laxatives could cause diarrhea, which can lead to metabolic acidosis. Ingestion of high amounts of salicylate acid can lead to toxicity and the development of metabolic acidosis. D) Adequate nutrition is necessary to prevent the buildup of acids in the blood. Incorrect administration of medication could cause a metabolic problem in the client with diabetes. The use of laxatives could cause diarrhea, which can lead to metabolic acidosis. Ingestion of high amounts of salicylate acid can lead to toxicity and the development of metabolic acidosis.

The healthcare provider prescribes digoxin for a client who will be discharged in the morning. When documenting the order in the medical record, which action by the nurse is most appropriate? A) Entering "digoxin, .0125 mg QD" B) Entering "digoxin, 0.0125 mg QD PO" C) Entering "digoxin, 0.0125 mg, once daily by mouth" D) Entering "digoxin, 1 pill each day"

C) Although many healthcare facilities supply an approved list of abbreviations and symbols, the nurse can best prevent confusion by writing out the order in full. This means the nurse should place a zero before the decimal when recording the dosage. The nurse should also write out "once daily" instead of using the abbreviation "QD," which may be interpreted incorrectly. Similarly, writing "by mouth" is preferable to using the abbreviation "PO." Simply recording "1 pill each day" is not appropriate because it does not specify a dosage amount.

The nurse is caring for a client who has been admitted to the unit with respiratory failure and respiratory acidosis. Which data from the nursing history would the nurse suspect contributed to the client's current state of health? A) Use of ibuprofen for the control of pain B) A recent trip to South America C) Aspiration pneumonia D) Recent recovery from a cold virus

C) Aspiration of a foreign body and acute pneumonia would put the client at risk for respiratory acidosis. A recent trip to South America would not constitute a respiratory risk factor. Recent recovery from a cold would not likely put the client at risk. Ibuprofen does not pose a threat to the respiratory health of the client.

A nurse on the behavioral health unit is leading a group regarding risk factors for anxiety. At the completion of group work, which comment made by a client would indicate the need for further teaching? A) "A lack of social interaction places me at risk for anxiety." B) "My personality could place me at risk for anxiety because I am shy." C) "Chronic illness is not a risk factor unless I am also unemployed." D) "I experienced a traumatic event that placed me at risk for having this anxiety disorder."

C) Chronic illness is a risk factor for anxiety disorders with or without the unemployment factor. For some clients multiple stressors, such as chronic illness with loss of employment, are risk factors. So this statement indicates a need for further teaching. The other statements are accurate and therefore do not require further teaching.

The nurse educator is teaching a group of nursing students about the purposes of documentation and medical records. Which of the following purposes is not appropriate for the educator to include in the teaching session with the students? A) Communication B) Planning C) Employee discipline D) Research

C) Client records are kept for a number of purposes. These include communication among healthcare professionals who are treating the same client, planning client care to evaluate care plan effectiveness, tracking services provided for reimbursement purposes, research to help all clients, education for students in health disciplines, healthcare analysis to determine agency needs, and quality assurance purposes. Although recordkeeping provides necessary legal evidence of the care provided and can thus be referenced during the employee discipline process, employee discipline is not a primary purpose of the medical record.

A newborn with pyloric stenosis has symptoms of projectile vomiting, leading to significant weight loss, dehydration, and metabolic alkalosis. What client teaching is necessary for the parents in caring for the infant until surgery to correct the defect? A) Monitoring for hyperventilation to detect changes in health status B) Breastfeeding techniques to reverse weight loss and dehydration C) Positioning of the infant to prevent aspiration D) Performing percussion and postural drainage to clear the airways

C) Complications related to aspiration of vomitus can be prevented by correct positioning of the infant. Parents should be taught correct positioning so they can care for the infant at home or when the nurse is not in the room. Hyperventilation leads to respiratory alkalosis and is usually unrelated to pyloric stenosis. Because pyloric stenosis blocks passage of food from the stomach to the small intestines, increasing oral intake will not be beneficial to the infant, so breastfeeding techniques are irrelevant at this time. Percussion and postural drainage are more relevant to lung diseases such as cystic fibrosis, not a gastrointestinal (GI) disorder such as pyloric stenosis.

During an assessment, the nurse becomes concerned that a client is at risk for developing metabolic alkalosis. What did the nurse assess that caused this concern? A) Daily ingestion of a banana with breakfast B) Daily weight consistent C) Daily use of sodium bicarbonate for gastric upset D) Daily use of prescribed nonsteroidal anti-inflammatory drugs (NSAIDs) for arthritic pain

C) Excess bicarbonate usually occurs as a result of ingesting antacids that contain bicarbonate, such as soda bicarbonate or Alka-Seltzer. Daily ingestion of a banana would prevent the development of hypokalemia from the daily use of sodium bicarbonate. Consistent daily weights would indicate fluid balance. Daily use of NSAIDs would not support the development of metabolic alkalosis.,

Use of flow sheets would be most appropriate during which phase of the nursing process? A) Evaluation B) Diagnosis C) Implementation D) Planning

C) Flow sheets use specific assessment criteria in a particular format. They are frequently used on a client's chart to record routine nursing tasks and assessment data. Examples of flow sheets include a graphic record, fluid balance record, daily nursing assessments record, patient teaching record, patient discharge record, and skin assessment record. Typically, flow sheets are used during the assessment and implementation portions of the nursing process.

A client tells a nurse that he believes he has an anxiety disorder because his mom and sister both have anxiety disorders. The nurse recognizes that the client believes in which theory related to the etiology of anxiety disorders? A) Neurochemical theories B) Neurobiological theories C) Genetic theories D) Humanistic theories

C) Genetic theories state that genetic predisposition plays a part in the development of anxiety disorders, with first-order family members being at higher risk of developing an anxiety disorder. Humanistic theories state that multiple factors contribute to the development of anxiety disorders. Neurochemical theories state that anxiety disorders are related to a disruption in neurotransmitter regulation. Neurobiological theories state that anxiety disorders are related to specific areas of the brain.

Decreased level of consciousness in acute respiratory acidosis is often due to hypercapnia causing: A) decreased pulse rate. B) hyperventilation. C) cerebral vasodilation. D) neurotransmitter disturbances.

C) Hypercapnia causes cerebral vasodilation, which results in headache, blurred vision, irritability, mental cloudiness, and decreased level of consciousness. The pulse rate is elevated in acute respiratory acidosis, not decreased. Respiratory acidosis is caused by hypoventilation, not hyperventilation. Neurotransmitter disturbances are unrelated to respiratory acidosis.

A child with acute asthma has a PaCO2 of 48 mmHg, a pH of 7.31, and a normal HCO3 blood gas value. The nurse interprets these findings as indicative of which condition? A) Metabolic acidosis B) Respiratory alkalosis C) Respiratory acidosis D) Metabolic alkalosis

C) If the pH is decreased and the PaCO2 is increased with a normal HCO3, it is uncompensated respiratory acidosis. Uncompensated respiratory alkalosis has an increased pH, decreased PaCO2, and normal HCO3. Uncompensated metabolic acidosis has a decreased pH, normal PaCO2, and decreased HCO3. Uncompensated metabolic alkalosis has an increased pH, normal PaCO2, and increased HCO3.

Terrorists have detonated a bomb in the downtown area of a major city, destroying part of a hotel, damaging nearby buildings, and killing or injuring an unknown number of people. A nurse in an emergency department handling many clients injured in the explosion receives a phone call from the babysitter for her children aged 5, 6, and 9, who says she's been watching the event unfold on TV since it happened. The nurse's family lives in a suburb more than 20 miles from the downtown. What should the nurse say? A) "Explain to the children that the people who did this are sick and will be punished for what they've done." B) "Please continue to watch the TV coverage with my kids in the room to ensure that you know what's going on at all times." C) "Please tell my children I'm alright but turn off the television and play a game with them to get them thinking about something else." D) "If you plan to continue to watch the TV coverage, please do it on the TV in my bedroom away from the kids."

C) Limit the amount of media and news coverage children are exposed to, as this can be frightening to younger children. In this case, it would be best for the babysitter to engage the children in play that does not involve obsessively watching the news. Continuing to watch the news nonstop, trying to explain the tragedy in terms of crime and punishment, or watching the TV away from the children is not appropriate.

A nurse is caring for a group of clients who are recovering in a rehabilitation hospital following total hip replacements. Which client is exhibiting the highest motivation to learn? A) A client who has been there the longest and is a great "coach" for newcomers B) A client who has been struggling with following nursing directives regarding discharge goals C) A client who is excited to learn ambulation techniques D) The client who has just moved in and is already eager for discharge

C) Motivation is the desire to learn and influences how quickly and to what extent an individual learns. It is generally greatest when an individual recognizes a need and believes the need will be met through learning. The client who is excited to learn about ambulation techniques understands that learning about it will help take his recovery to a high level. Motivation must be experienced by the client, not by someone else (as in being a "coach" for newcomers). Clients who struggle with rules or following prescribed courses of treatment are not motivated to learn the best reason for their particular plan of action; they may be "bucking" the system. The client who is already waiting to go home may be eager for that to occur, but not necessarily to the extent of being ready to learn how to achieve this end.

If more older adults live in Mississippi than elsewhere in the United States and clients in Massachusetts have much greater access to health services than clients elsewhere in the United States, then what does this imply about access of older adults to healthcare in Mississippi? A) The likelihood is that more specialists serving older populations will work in Mississippi than in Massachusetts. B) Their access to healthcare should be roughly equal to that of older adults living in Massachusetts but with a different mix of providers. C) The need for services will be much greater for older adults in Mississippi than in Massachusetts due to a decreasing number of healthcare providers. D) There will be much more robust rural services for older adults in Mississippi than in Massachusetts.

C) No states have enough primary care providers to meet their needs. Aggravating this problem is the growing number of healthcare providers who specialize. If Massachusetts is a state with a much greater than average access for clients to health services, it is likely that states such as Mississippi offer less access to these services and fewer primary care physicians, and Mississippi's greater population of older adults will lack access to the care they need. Nothing here implies that there will be fewer specialists working in Massachusetts, and the trend is for more specialists everywhere. Nothing here indicates anything about the mix of providers in either state, and rural services tend to be lacking

A client is admitted to the emergency department (ED) for treatment of an overdose. The client's arterial blood gas results indicate acute respiratory acidosis. Which substance found on the nurse's review of the toxicology analysis is most likely the cause for the client's current condition? A) Cocaine (a stimulatory anesthetic) B) Marijuana (a cannabinoid) C) Oxycodone (a narcotic) D) PCP (a dissociative anesthetic)

C) Oxycodone is an opiate narcotic. Excessive use or overdose of narcotic substances can lead to respiratory depression and respiratory acidosis. Cocaine is a stimulant. Marijuana does not depress the central nervous system or respiratory center. PCP is a hallucinogenic agent.

What is a good way for a nurse to prepare the environment for teaching? A) Keep to a strict schedule decided in advance. B) Emphasize the importance of paying close attention if client reactions demonstrate confusion. C) Evaluate client abilities to perform skills with return demonstrations. D) Inform students that they need to take effective notes because you will not be repeating yourself

C) Return demonstrations are an effective way to evaluate whether clients are able to perform newly learned skills, and they contribute to a good environment for teaching. Nurses should take time for teaching, not simply adhere to strictly predefined schedules. Nurses must be able to adapt teaching based on client reactions, and they should be prepared to repeat instructions.

The client is receiving sodium bicarbonate intravenously (IV) for correction of acidosis secondary to diabetic coma. The nurse assesses the client to be lethargic, confused, and breathing rapidly. Which is the nurse's priority response to the current situation? A) Stop the infusion and notify the physician because the client is in alkalosis. B) Decrease the rate of the infusion and continue to assess the client for symptoms of alkalosis. C) Continue the infusion, because the client is still in acidosis, and notify the healthcare provider. D) Increase the rate of the infusion and continue to assess the client for symptoms of acidosis.

C) The client receiving sodium bicarbonate is prone to alkalosis; monitor for cyanosis, slow respirations, and irregular pulse. The client's symptoms do not indicate alkalosis so infusion should not be stopped. The client continues to exhibit signs of acidosis; symptoms of acidosis include lethargy, confusion, CNS depression leading to coma, and a deep, rapid respiration rate that indicates an attempt by the lungs to rid the body of excess acid, and the physician should be notified. The infusion should not be increased or decreased without a practitioner order.

During a home visit, the nurse evaluates teaching provided to a client recently hospitalized for metabolic alkalosis. Which observation indicates that additional teaching is required? A) Drinks 2 cups of black coffee each day. B) Consumes one orange each day with breakfast. C) Ingests bicarbonate of soda after each meal. D) Monitors and tracks daily weights.

C) The indiscriminate ingestion of sodium bicarbonate is a risk factor for the development of metabolic alkalosis. Black coffee is not associated with the development of metabolic alkalosis. Oranges contain potassium, which is beneficial to prevent the development of metabolic alkalosis. Tracking of daily weights would help detect a fluid imbalance, which is associated with metabolic alkalosis.

The nurse is conducting a class for a group of pregnant clients and wants to focus specifically on the risks of alcohol consumption for the developing fetus. Which topic should the nurse include with regard to safety of the fetus? A) Human growth and development B) Nutrition C) Lifestyle modification D) Stress management

C) The nurse should focus on lifestyle modification if she plans to focus specifically on the risks of alcohol consumption for the developing fetus. Human growth and development, nutrition, and stress management are all worthwhile topics for these clients, but none of them as directly address the risks of alcohol consumption.

The nurse is caring for a client who is being mechanically ventilated. The current ventilator settings are: respiratory rate, 25 breaths per minute; tidal volume, 600 mL; FiO2, 30%; humidification 30 mg H2O/L. After being ventilated for 2 hours, arterial blood gas analysis reveals a pH of 7.20 and a PaCO2 of 49 mmHg. Which change in ventilator settings should the nurse anticipate? A) Increase in humidification of inspired air B) Decrease of FiO2 from 30% to 25% C) Increased respiratory rate to 30 breaths per minute D) Decreased tidal volume of each breath

C) This client is exhibiting respiratory acidosis that is not corrected by the current ventilator settings. This client needs to "blow off" more CO2; therefore, the respiratory rate would be increased. Both decreasing the FiO2 and decreasing the tidal volume would decrease the amount of CO2 expelled. Humidification has no effect on the amount of CO2 expelled.

An adolescent is hospitalized following several days of vomiting due to food poisoning. The nurse is planning to include which points when teaching the client's family at discharge? Select all that apply. A) Immunizations for the adolescent B) Nutritional patterns of the adolescent C) Signs and symptoms of metabolic alkalosis D) Proper food-handling techniques E) Normal laboratory values of the adolescent

C, D) The family of anyone experiencing prolonged vomiting should be taught the signs and symptoms of metabolic alkalosis. In this case, the nurse would include teaching about proper methods of food handling to prevent further episodes of food poisoning. Food patterns of the adolescent are not the precipitating factor of the food poisoning, and immunizations would not prevent this disease. Unless the family asks, it is not necessary to teach normal laboratory findings.

A nurse educator is teaching a group of students about managed care. The educator knows that the students have understood the concept when they state that managed care has which emphasis? Select all that apply. A) Bringing services of multiple providers to the client B) Organizing healthcare services around the stated needs of the client C) Cost-effective care D) Preventive services E) Health promotion

C, D, E) Managed care describes a healthcare system that emphasizes cost-effective, quality care that focuses on decreased costs and improved outcomes for groups of clients. Managed care clinics will emphasize cost-effective care by offering preventive services and health promotion activities. Case management describes a range of models for integrating and delivering healthcare services from multiple providers to the client. Client-focused care is a delivery model that organizes healthcare services around the stated needs of the client

The nurse is planning care for the client who has been admitted with metabolic alkalosis. Which are appropriate nursing diagnoses for this client during the acute phase of the illness? Select all that apply. A) Ineffective Health Maintenance B) Risk for Hypothermia C) Deficient Fluid Volume D) Risk for Impaired Gas Exchange E) Risk for Injury

C, D, E) Respiratory compensation for metabolic alkalosis includes depression of the respiratory rate and reduction of the depth of respirations, leading to the retention of carbon dioxide. Patients with metabolic alkalosis often have an accompanying fluid volume deficit. With the fluid volume deficit, the client would experience hyperthermia. Ineffective health maintenance would not be a priority during the acute phase of the disease but, rather, a teaching opportunity before discharge depending on the cause of the metabolic alkalosis. The client is at risk for injury because of the associated muscle spasms and dizziness.

The nurse identifies the diagnosis Risk for Injury as appropriate for a client with metabolic acidosis. Which strategies should the nurse use to support this diagnosis? Select all that apply. A) Apply wrist restraints and secure to the bed frame. B) Discuss chemical restraint use with the healthcare provider. C) Keep the bed in the lowest position. D) Keep bed side rails raised. E) Place a clock and calendar at the bedside.

C, D, E) To reduce the client's risk for injury, the nurse should make sure the bed is kept in the lowest position and the side rails are raised. A clock and calendar at the bedside will help with orientation. Restraints are used in the event the client demonstrates harm to self or others. Confusion or a risk for injury is not a reason to use wrist or chemical restraints.

The nurse is reviewing new orders written for a client experiencing respiratory alkalosis. Which orders would be appropriate for this client's care needs? Select all that apply. A) Oxygen 2 liters via face mask B) Restrict fluids to 2 liters per day. C) Admit to a private room. D) Infuse 1 ampule of sodium bicarbonate now. E) Draw arterial blood gases.

C, E) The client has respiratory alkalosis, which is caused by hyperventilation. Additional oxygen is not required. A fluid restriction is not required in the treatment of respiratory alkalosis. Management of respiratory alkalosis focuses on correcting the imbalance and treating the underlying cause. It is important to create a calm, quiet, low-stimulation environment to reduce the client's anxiety or panic. Sodium bicarbonate is used in the treatment of respiratory and metabolic acidosis. Arterial blood gases must be ordered prior to beginning medication or oxygen therapy.

The nurse is reviewing the latest arterial blood gas results for a client with metabolic alkalosis. Which result indicates that the metabolic alkalosis is compensated? A) pH 7.32 B) PaCO2 18 mmHg C) HCO3 8 mEq/L D) PaCO2 48 mmHg

D) A normal pH level is 7.35-7.45. A pH of less than 7.35 is acidosis. A PaCO2 level of 18 mmHg is low and is seen in respiratory alkalosis. A HCO3 level of 8 mEq/L is low and is most likely associated with metabolic acidosis. In metabolic alkalosis, there is an excess of bicarbonate. To compensate for this imbalance, the rate and depth of respirations decrease, leading to retention of carbon dioxide. The PaCO2 will be elevated.

A client states, "I haven't left my house for 6 years." Based on this data, which diagnosis does the nurse anticipate for this client? A) Hematophobia B) Social anxiety disorder C) Pathophobia D) Agoraphobia

D) Agoraphobia is characterized by anxiety associated with two or more of the following situations: being in enclosed spaces, being in open spaces, using public transportation, being in a crowd or standing in a line of people, or being alone outside the home environment. Social anxiety disorder is a fear of one or more social situations that may lead to scrutiny by others. Hematophobia is the fear of blood. Pathophobia is the fear of disease.

The detonation of several incendiary devices in a suburban area has caused widespread fires. A nursing home nearest one of the largest fires needs to be evacuated. What is the concern a nurse working in the home will have for his clients in this situation? A) Socioeconomic limitations B) Diminished sensory awareness C) Inadequate thermoregulation mechanisms D) Limited mobility

D) Although diminished sensory awareness, inadequate thermoregulation mechanisms, and socioeconomic limitations all must be considered for older adults in an emergency, the primary issue that should concern the nurse in this situation is his clients' limited mobility. They need to evacuate, and doing so quickly could be difficult for many of his clients.

The results of a client's arterial blood gas sample reveal an oxygen level of 72 mmHg. For which associated health problem should the nurse assess this client? A) Stress and coping B) Perfusion C) Fluid and electrolyte imbalance D) Cognition

D) An oxygen level of less than 75 mmHg can be due to hypoventilation. This drop in oxygen will change the client's level of responsiveness. Perfusion is affected by a reduction in circulating fluids. With a fluid and electrolyte imbalance, there is another disorder affecting acid-base balance. This might not be affected by oxygen level. Stress and coping may need to be analyzed to determine how the client is coping with the anxiety related to low oxygen levels, but this is not directly a physiological health problem.

An older adult client has a history of heart disease and dementia and takes several medications. His wife states that sometimes he forgets to take his medications, or he takes multiple doses of his medications, due to his dementia. An accidental overdose of which medication could result in metabolic acidosis? A) Losartan (an angiotensin II receptor blocker to reduce hypertension) B) Simvastatin (a statin to reduce blood cholesterol levels) C) Rivastigmine (a cholinesterase inhibitor to reduce symptoms of dementia) D) Aspirin (a salicylate to decrease risk of heart attack)

D) Aspirin is salicylic acid, which could decrease the blood pH if taken in high quantities. Diuretics, some antidepressants, antiseizure medications, and angiotensin-converting enzyme (ACE) inhibitors could all affect the acid-base balance in an older adult, but acid-base balance is less affected by angiotensin II receptor blockers, statins, and cholinesterase inhibitors.

When considering acid-base balance, health promotion should focus on A) conducting yearly health screenings. B) obtaining immunizations. C) beginning an exercise regimen. D) maintaining fluid balance.

D) Both overhydration and dehydration can result in acid-base imbalances. Therefore, health promotion should focus on maintaining fluid balance. Beginning an exercise regimen, obtaining immunizations, and conducting yearly health screenings are activities that can promote health in other areas.

A 2-month-old infant has been diagnosed with pneumonia with respiratory alkalosis. The provider also suspects that the infant is suffering from paresthesias of the hands and feet, because the infant pulls away and cries when his extremities are touched. What client teaching can the nurse provide the parents to comfort the infant with paresthesias? A) Postural drainage techniques B) Massage techniques C) Breastfeeding techniques D) Swaddling techniques

D) Comfort measures that may be successful for infants experiencing paresthesias include swaddling, calming touch, and speaking with a quiet voice. A mother of a 2-month-old infant likely already understands breastfeeding techniques if she breastfeeds the infant. Massage techniques will likely cause additional discomfort for the infant; calming touch is needed instead. Postural drainage techniques are used to clear the airway, not comfort the infant.

Occupation-specific stressors that are ongoing and unmanaged can lead to what extreme form of stress? A) Distress B) Eustress C) Allostasis D) Burnout

D) Eustress is good stress that leads to accomplishment and victory. Distress is bad stress that is associated with inadequacy, insecurity, and loss. Although occupation-specific stressors can be a type of distress, distress is not the specific term for the extreme form of stress caused by ongoing and unmanaged stress. The term used for that form of extreme stress is burnout. Burnout in nurses can lead to reduced quality of care and decreased patient satisfaction. Allostasis refers to the changes necessary to achieve homeostasis.

Why should a nurse take daily weights of a client with acid-base balance? A) It helps monitor oxygenation status. B) It helps monitor perfusion of organs. C) It helps monitor renal function. D) It helps monitor fluid balance.

D) Fluid balance must be maintained to support acid-base balance. If a client rapidly gains weight, it is a sign of fluid overload. If a client rapidly loses weight, it is a sign of dehydration. Both of these conditions can alter the acid-base balance, so a client's weight should be monitored daily. A client's weight does not reflect oxygenation status or perfusion of organs. Daily weights can reflect renal function, but weight can fluctuate even if the kidneys are functioning properly.

The nurse is reviewing prescriptions written for a client with chronic respiratory acidosis. Which prescription should the nurse question prior to implementation? A) Keep head of the bed elevated to 40-degree angle. B) Dextrose 5% and 0.45% normal saline at 100 mL per hour C) Consult Respiratory Therapy for breathing treatments four times a day. D) Oxygen 6 liters per minute per nasal cannula

D) In clients with chronic respiratory acidosis, oxygen is administered cautiously to prevent carbon dioxide narcosis. Adequate hydration such as intravenous fluids is important to promote removal of respiratory secretions. Pulmonary hygiene measures such as breathing treatments may be instituted. Elevating the head of the bed promotes oxygenation.

Handoff communication, or the transfer of data during transitions in care, includes an opportunity to ask questions, clarify, and confirm the information being passed between sender and receiver. What is the main objective for ensuring effective communication during a client handoff? A) To avoid lawsuits B) To make sure all documentation is complete C) To facilitate quality improvement D) To ensure client safety

D) Ineffective communication is the primary cause of sentinel events, making client safety the primary objective of the handoff communication process. Handoff communication may be scrutinized during a lawsuit, but avoiding litigation is not a primary objective. Similarly, engaging in handoff communication can help a nurse determine whether all documentation related to a particular client's care is complete, but this is not a primary objective. Finally, analysis of handoff communication may be a quality improvement criterion, but it is not a primary objective.

A nurse, who works in a clinic environment, places great emphasis on cost control, customer satisfaction, health promotion, and preventive services. The primary focus of this nurse is representative of which type of healthcare system? A) Functional method B) Client-focused care C) Case method D) Managed care

D) Managed care describes a healthcare system whose goals are to provide cost-effective, quality care that focuses on decreased costs and improved outcomes for groups of clients. The case method of care is a client-centered model in which one nurse is assigned to care for a group of clients during an 8- to 12-hour shift. The functional method of care focuses on jobs to be completed and is task oriented. Client-focused care is a delivery model that brings all services and care providers to the client.

The nurse is caring for a client who sustained multiple injuries in an automobile accident. As a part of secondary prevention for this client, which does the nurse include in the plan of care? A) Promote wellness. B) Detect early disease. C) Restore the client to previous functioning. D) Prevent the progression of more symptoms.

D) Rehabilitation is tertiary prevention and is aimed at restoring the client to the previous level of functioning. Prevention of the progression of symptoms and early detection of disease are secondary preventions. Promoting wellness is considered primary prevention.

A novice nurse is looking for employment and hopes to find a facility where the nursing staff participates in making, implementing, and evaluating client care policies. Which organizational module implements these practices? A) Client-focused care B) Differentiated practice C) Managed care D) Shared governance

D) Shared governance is an organizational model in which nursing staff are cooperative with administrative personnel in making, implementing, and evaluating client care policies. Differentiated practice is a system in which the best possible use of nursing personnel is based on their educational preparation and resultant skill sets. This model consists of specific job descriptions for nurses according to their education or training. Client-focused care is a delivery model that brings all services and care providers to the client. Managed care focuses on cost containment, consumer satisfaction, health promotion, and preventive services.

A nurse conducted a safety class for a group of older adult clients in the community on fall prevention. During a follow-up visit in the home of one of these clients, the nurse sees a number of fall hazards she identified during her class. What should the nurse document regarding the learning outcome for this client? A) The client might not have heard necessary information during the class. B) The client did not understand the teaching given in the class. C) The client understood the teaching given in the class but chose to ignore it. D) The client did not comply with the teaching given in the class.

D) The client did not implement what he was taught in class given the evidence of fall hazards the class specifically addressed. The nurse should document the learning outcome as noncompliance. The client might not have heard necessary information during the class because of a hearing deficit or being distracted, the client might not have understood the information because of the way it was presented or for some other reason, or the client may have chosen not to comply with the teaching, but the nurse cannot know which of these might be the case, if any, without further evaluation of the client.

The nurse is caring for a client who has been diagnosed with diabetes mellitus. The client must learn how to independently perform fingerstick blood sugar analysis as part of the plan of care. The client says, "I already know what you are attempting to teach because I looked everything up on the internet." Which is the best action by the nurse based on the client's statement? A) Document that the client understands teaching. B) Teach the client's support system how to perform the procedure. C) Give the client printed learning materials. D) Watch the client perform a return demonstration of the skill.

D) The nurse is responsible for documenting that the client can perform the skill that has been taught. Giving the client written directions or teaching the support individual does not meet the requirement that the client will perform the skill. The nurse cannot document that the client understands teaching until a return demonstration by the client is correctly performed.

The nurse instructs a client with a history of acute respiratory acidosis and lung infections on ways to prevent further episodes of the health problem. Which client statement indicates that teaching has been effective? A) "I will limit drinking alcohol to the evening hours only." B) "I will limit my intake of bananas and oranges." C) "I will take prescribed antibiotics until my symptoms subside." D) "I will receive the annual influenza vaccination."

D) The nurse should discuss ways to avoid future episodes of acute respiratory infections by encouraging the client to receive immunization against pneumococcal pneumonia and influenza. Alcohol is a central nervous system depressant, which can adversely affect respiratory status and lead to the development of respiratory acidosis. The ingestion of bananas and oranges will not promote the development of respiratory acidosis. The client should be instructed to complete a full course of antibiotics prescribed to treat infections.

The nurse is managing the care of an older adult client who is near the end of life but still has treatment options available. The treatment options could potentially be painful and expensive, and they would only extend the client's life approximately 2 months. What important topic should the nurse include during initial discussions with the client and family? A) The need for hospice care for the client B) Referral for grief counseling for the family C) Reasons why the client should remain in the hospital until death D) The client's health and impending mortality

D) When providing case management for a client near the end of life, the case manager's first step in ensuring the client's wishes are met is to speak honestly to the client about his or her health and impending mortality. Hospice care should be presented as an option, but only if the client chooses not to receive additional treatment. The client should be given the choice of where he or she wants to die, not be encouraged to remain in the hospital until death. Grief counseling for the family would come at a later appointment, not during initial discussions

A nursing student has been assigned to present a teaching project to the class, using each of Bloom's taxonomy domains. The student has planned several activities to include when presenting the project to the class. Which activities are within the affective domain? Select all that apply. A) Class members must read a paragraph about a new clinical trial, summarize the information, and present it to the rest of the class. B) Class members must list the technical skills they have learned. C) Class members must demonstrate a favorite nursing skill for the class. D) Class members must reflect on how they felt the first time they provided direct client care. E) Class members must identify two attitudinal changes that have occurred in their lives since beginning their nursing education.

D, E) In cognitive theory, learning occurs across three primary domains: cognitive, or "thinking"; affective, or "feeling"; and psychomotor, or "skill." The affective domain includes emotional responses to tasks, such as feelings, emotions, interests, attitudes, and appreciations. Listing technical skills and reading or summarizing information is part of the cognitive domain, which includes knowing, comprehending, application, analysis, synthesis, and evaluation. The psychomotor domain is the "skill" domain and includes hands-on motor skills, such as demonstration.

A client asks the student nurse to explain the pathophysiology of diabetes. The student nurse does not know the answer to this question. What should the student respond to the client? A) "I do not know, but I will find out." B) "You'll have to ask the doctor that question." C) "Why do you need to know that?" D) "I do not know."

A) The student nurse best exhibits integrity and awareness of self-limits by admitting not knowing the answer, but offering to find out and get back to the client. Honesty by the student nurse will build trust with the client. Just saying "I don't know" and leaving it at that will make the nurse appear untrustworthy and possibly uncaring. Questioning the client about the need for the information is defensive and might also be construed as a challenge by the client. Referring the question to the doctor is also not good ethics; nurses need to continue learning throughout their practice.

Which hormone is one of the primary mediators of stress? A) Glucagon B) Cortisol C) Serotonin D) Somatostatin

B) The two primary stress mediators are glucocorticoids (e.g., cortisol) and catecholamines (e.g., epinephrine). Serotonin is a neurotransmitter that is involved in some mood and anxiety disorders, but it is not a primary mediator of stress. Somatostatin is a hormone released by the pituitary gland. It is not involved in the stress response. Glucagon is secreted by the pancreas to increase blood glucose levels.

The nurse is caring for a client who was admitted to the emergency department with abdominal pain. The client speaks very little English and requires an emergency appendectomy. The nurse has enlisted the hospital interpreter to explain the procedure and help with informed consent. When the interpreter arrives, which action by the nurse is appropriate? A) Ask the interpreter to translate as closely as possible. B) Ask the client's family to be included in the interpreting process and exchange of information. C) Direct questions to the interpreter and not the client. D) Request that the interpreter use the same dialect as the client to promote understanding.

A) An interpreter is an individual who mediates spoken or signed communication between people who use different languages without adding, omitting, distorting meaning, or editorializing. It is not the interpreter's responsibility to determine the dialect with which the client is most familiar. The nurse should direct all questions to the client, not the interpreter. The nurse should also avoid asking the client's family, especially a child or spouse, to help interpret.

The nurse observes a client being treated for depression sitting with the head down and avoiding conversation with peers. Which nursing intervention is most appropriate for this client? A) Ask open-ended questions about the client's feelings. B) Ask the client closed-ended questions. C) Encourage a peer to sit with the client and the nurse. D) Tell the client that lack of involvement leads to more depression.

A) An open-ended question encourages more than a one-word response. Depressed clients should be comfortable with a one-to-one interaction prior to other client involvement. A closed-ended question is unlikely to encourage continued communication. Telling the client that if he does not get involved he will become more depressed is not encouraging communication.

A client who has attempted to commit suicide in the past tells the nurse about feeling better since being prescribed an antidepressant medication. Which conclusion by the nurse is appropriate based on the assessment data? A) Improved mood B) Improved sleep C) Improved feelings of guilt D) Improved appetite

A) Antidepressants treat major depression by enhancing mood. Antidepressants are also prescribed to treat anxiety disorders. Recent studies link depression and anxiety to similar neurotransmitter dysfunction, and both seem to respond to treatment with antidepressant medications. An improvement in appetite and sleep might occur with antidepressant medication; however, the medication does not directly stimulate the appetite or cause sleep. Antidepressant medication is not known to improve feelings of guilt.

A nurse educator in a medical-surgical unit is demonstrating the use of new equipment to the rest of the nurses on the unit. After initial efforts at having the class gather closely around the models were met with discomfort and inattention, the nurse educator sets up the models in the front of the classroom. Which level of proxemics would be ideal for this situation? A) 4 to 12 feet B) 1 1/2 to 4 feet C) 12 to 15 feet D) Less than 1 1/2 feet

A) Because the nurse is communicating with a group and all members of the group must be able to see the models clearly, a social distance of 4 to 12 feet would be appropriate. Social distance is characterized by a clear, visual perception of the whole individual, and it is expedient for communicating with several people at the same time. In contrast, an intimate distance of 1 1/2 feet or less would likely make the group members extremely uncomfortable and prevent them from seeing all aspects of the demonstration. A personal distance of 1 1/2 to 4 feet might still cause discomfort among participants and make it difficult for them to observe the models in full. Finally, a public distance of 12 to 15 feet would make it hard for participants to observe the models' smaller features and would also make the communication process less personal for all parties involved.

For a client with obsessive-compulsive disorder with contamination obsessions, what nursing assessment is essential to development of an effective client care plan? A) Assessment for skin integrity B) Assessment for sexual activity C) Assessment for tics D) Assessment for religious beliefs

A) Clients with contamination obsessions often have compulsions related to cleaning and washing, especially washing the hands. Continual exposure to water and cleansing agents may result in loss of skin integrity. Assessment for sexual activity or religious beliefs would be more important if the client had aggressive, sexual, or religious obsessions. Assessment for tics would be more important for individuals with symmetry obsessions.

When caring for a client newly diagnosed with obsessive-compulsive disorder, which action by the nurse is appropriate? A) Do not interrupt the ritual. B) Interrupt the ritual. C) Teach about antianxiety foods. D) Teach ritual interruption skills.

A) Do not interrupt the ritual because the client may feel compelled to start from the beginning. For the newly diagnosed client, teaching ritual interruption skills and teaching about antianxiety foods would not be the priority.

Which is the priority nursing action when providing care to a client who demonstrates signs of escalating anxiety? A) Isolate the client in a safe, quiet, and protective environment. B) Leave the client alone in a room. C) Provide a benzodiazepine. D) Phone the physician.

A) The nurse should first isolate the severely anxious or panicked client in a safe, quiet, protective environment. The nurse should not leave the client unattended. Phoning the physician may not be helpful to the client. Medications can be provided once the client is in a safe, protective environment.

A client worries every day about personal health and states, "I may not have enough medication if the weather takes a turn for the worse." This client is exhibiting a sign of which alteration in stress and coping? A) Generalized anxiety disorder B) Phobia C) Obsessive-compulsive disorder D) Panic disorder

A) Generalized anxiety disorder is excessive worry about everyday problems, with the anxiety being more intense than the situation warrants. The client is demonstrating signs of generalized anxiety disorder. A phobia is an intense, persistent, irrational fear of a simple thing or social situation that compels the individual to avoid the stressor that elicits the fear. Panic disorder is a sudden attack of terror, accompanied by a pounding heart, sweatiness, weakness, faintness, or dizziness. Obsessive-compulsive disorder is characterized by obsessive thoughts and compulsive repetitive behaviors formed in response to the obsessive thoughts to lower the level of anxiety experienced.

A nurse is providing care to a client who is scheduled for a colonoscopy. The client requires a bowel prep prior to the diagnostic test. Which approach should the nurse use to facilitate the client's understanding of the procedure? A) Use layman's terms to explain the procedure, then ask the client to describe the procedure in her own words B) Use medical terminology when explaining the procedure to the client to ensure maximum accuracy and clarity C) Focus on intonation when describing the procedure to the client D) Speak slowly and loudly when providing client teaching about the procedure

A) Good verbal communication incorporates simplicity, brevity, and completeness. Simplicity involves the use of commonly understood words rather than medical terminology. For example, the term "bowel prep" may be completely meaningless to the client, so telling the client that she needs to drink a gallon of laxative-like medication will get the point across better. Asking the client to repeat the information back in her own words gives the nurse a chance to evaluate whether the teaching has been successful. While intonation can modify the feeling and impact of a message, focusing on intonation is less important than using easily understood terminology. Speaking too slowly or too loudly could be interpreted by the client as patronizing or aggressive.

The urgent care clinic nurse is treating a client who is experiencing abdominal pain. The client states, "I think I ate tainted food last night." What should the nurse do after the client states that the food was tainted? A) Ask the client open-ended questions to further assess the situation. B) Tell the client the healthcare provider does not need to assess the client. C) Call an ambulance before assessing the client any further. D) Advise the client to take an antacid.

A) In problem solving, the nurse obtains information that clarifies the nature of the problem. This is the first step. The nurse cannot make decisions such as calling an ambulance or telling the client not to see a healthcare provider before knowing all the essential information. The nurse should not give advice about taking a medication.

A nurse is interviewing a client who recently attempted suicide. Which question is appropriate for the nurse to ask the client? A) "Do you currently have a plan for killing yourself?" B) "Why would you think about harming yourself?" C) "Did you feel unsafe?" D) "Do you ever think about hurting yourself?"

A) It is helpful to recognize the client's current state, not a past state, by talking about depression and asking direct questions such as "Do you currently have a plan for killing yourself?" This is an effective type of questioning. The client has recently attempted suicide, so questions asking whether the client has thought about hurting himself deny the reality of the situation. Asking why the client would hurt himself could be interpreted as being judgmental and may damage the nurse—client relationship.

A client prescribed an antidepressant tells the nurse that the pill causes dizziness upon standing or changing position too quickly. This is a common side effect of which antidepressant medication? A) Atypical antidepressant B) Monoamine oxidase inhibitor (MAOI) C) Selective serotonin reuptake inhibitor (SSRI) D) Lithium

A) One side effect of atypical antidepressants is orthostatic hypotension, so clients should be instructed to change positions slowly, especially when moving from a lying to a sitting position or a sitting to a standing position. Orthostatic hypotension is not a common side effect of MAOIs or SSRIs. Lithium is used to treat bipolar disorder, not depression.

A nurse at a psychiatrist's office is reviewing the medication prescribed to several new clients for mood disorders. Which order would the nurse question? A) A prescription for paroxetine for a 15-year-old boy with depression B) A prescription for fluoxetine for a 14-year-old girl with depression C) A prescription for sertraline for a 10-year-old boy with obsessive-compulsive disorder D) A prescription for sertraline for an 11-year-old girl with depression

A) The FDA does not recommend paroxetine to treat depression in children and adolescents because of an increased risk in suicidal thinking and behavior during initial treatment. All other prescriptions are appropriate for the age and disorder.

Which finding would indicate that treatment for a client with obsessive-compulsive disorder is effective? A) The client watches television while eating meals and engages in conversation with a roommate. B) The client conducts ritualistic hand washing every hour. C) While walking, the client counts 13 steps and then reverses the direction and repeats the process. D) The client folds and refolds clothing in a drawer before each meal.

A) The client who watches television while eating meals and engages in conversation with a roommate is exhibiting behavior that suggests treatment for obsessive-compulsive disorder is effective. This behavior is evidence of reduced anxiety and less of a need to engage in ritualistic behavior. The other observations would indicate the need for additional treatment.

The home health nurse is visiting a client who is 2 weeks postoperative from a coronary artery bypass surgery. The client has lost 10 pounds, is continuing to experience pain, and is not eating. What should be the nurse's next action? A) Examine the current interventions for pain relief. B) Refer the client to social services. C) Contact Meals on Wheels so that the client will eat. D) Revise the goals in the current plan of care.

A) The nurse evaluates that pain goals for this client have not been met and examines pain relief interventions to determine the problem. The goal of pain management is pain relief, and that goal would not change; what might change is the interventions to meet the goal. Contacting Meals on Wheels may not be appropriate if the problem is pain relief. Pain relief is a medical issue that is addressed by the nurse and physician, not social services.

Which condition is associated with the highest rate of comorbidity with depression? A) Alcohol abuse B) Obesity C) Back problems D) Hypertension

A) The rate of comorbidity between mood disorders and substance abuse disorders is high. Approximately 20.5% of individuals who are alcohol dependent also have a major depressive disorder, and those who are alcohol dependent are 3.7 times more likely to have major depression than those who are not dependent on alcohol. Individuals with bipolar disorder have high rates of alcohol and substance abuse, and suicide is the leading cause of death among individuals with substance abuse disorders. Obesity, back problems, and hypertension do not share this same link with mood disorders.

A 72-year-old client presents to the clinic with complaints of restlessness, muscle tension, and increased perspiration. Her vital signs are P 112, R 23, BP 131/85, and T 97.8°F. The nurse recognizes these manifestations as signs and symptoms of moderate anxiety. However, the client reports that she does not feel anxious about anything and has never before been diagnosed with an anxiety disorder. What other factor must the nurse consider based on this client's age? A) These manifestations could instead be related to a medical illness. B) These manifestations could be related to an overdose of antianxiety medications. C) These manifestations could indicate a change in the client's cognitive functioning. D) These manifestations could be related to drug-drug interactions between selective serotonin reuptake inhibitors (SSRIs) and other medications.

A) The scenario indicates that the client has not been previously diagnosed with an anxiety disorder, so these symptoms are not likely due to an overdose of antianxiety medications or drug-drug interactions between SSRIs and other medications. The client also does not present with symptoms that indicate cognitive function is declining. The most likely explanation is that these symptoms are related to a medical illness other than anxiety, because manifestations of anxiety often overlap with manifestations of other medical illnesses in older adults.

Which intervention can the nurse implement independently when caring for a client with alterations in stress and coping? A) Therapeutic communication B) Cognitive-behavioral therapy C) Psychotherapy D) Administration of medications

A) Using therapeutic communication is an essential intervention that the nurse can implement independently when caring for a client with an alteration in stress and coping. Cognitive-behavioral therapy, psychotherapy, and administration of medications are all collaborative interventions.

After receiving the morning report, the nurse prioritizes care needed by several clients. Which factors should the nurse keep in mind when creating this priority list? Select all that apply. A) Client condition B) Safety C) Time available D) Client preferences E) Nurse preferences

A, B, C, D) Prioritization means more than just making decisions about which interventions to do first, second, or third. Factors that influence prioritization include client condition, safety factors, available time, and client preferences. A change in client status may very well require reevaluating priorities and changing the planned order of interventions. Nurses can be fair in their allocation of time, attention, and skills to ensure client safety. Time priorities are determined by the urgency of completing the interventions for the clients. The nurse should strive to honor the client's wishes as much as possible and complete nursing interventions as needed. The nurse needs to complete priority tasks whether or not the task is enjoyable, so nurse preferences are often set aside when creating a priority list of care needed by clients.

A client with depression is receiving electroconvulsive therapy (ECT). Which interventions should the nurse plan when caring for this client? Select all that apply. A) Maintain nothing-by-mouth status until fully awake. B) Administer intravenous fluids for 8 hours postprocedure. C) Place in the lateral recumbent position. D) Provide oral fluids immediately after the procedure. E) Place in the supine position with the head flat.

A, C) Care of the client recovering from electroconvulsive therapy includes placing in the lateral recumbent position to facilitate drainage and to prevent aspiration and to maintain nothing by mouth until fully awake. The supine with head flat position can lead to aspiration. The client does not need intravenous fluids for 8 hours after the procedure. Providing oral fluids when not fully awake can lead to aspiration.

The nurse manager is concerned that a staff nurse is having difficulty prioritizing client care needs. Which did the manager observe the nurse perform that caused these concerns? Select all that apply. A) Relying on another nurse's assessment B) Reviewing the medication administration record C) Not completing an assessment D) Doing easiest tasks first E) Asking unlicensed assistive personnel to perform complicated care

A, C, D, E) Obtaining assessment data from another nurse can provide insight and give a picture of how a client has been during the previous shift; however, using only this information to set priorities may negatively impact client outcomes. Failing to complete an assessment can cause the nurse to miss important and necessary information when setting priorities. Completing easy tasks before doing important, necessary tasks does not make good, professional common sense. Inappropriate delegation may result in the nurse having to do an intervention over or may even result in harm to the client. Reviewing the medication administration record does not negatively impact prioritization.

The nurse is taking the time to reflect on a care situation in which a client sustained a cardiac arrest and died. On which areas should the nurse focus when performing this reflection? Select all that apply. A) Things that could have been done differently B) Gut reactions to the situation C) Things that were done well D) Resources that were used at the time E) Resources that were needed but not available

A, C, D, E) Reflection is the action of making sense of occurrences, situations, or decisions by carefully considering the totality of the experience, such as what worked or did not work, what could have been done differently to achieve better outcomes, what was done well, and what necessary resources were available. In order to reflect on an experience, nurses need to learn what to pay attention to or notice. Reflective thinking can change a situation that is obscure, uncertain, and disturbing to one that is clear, understandable, and settled. A "gut reaction" describes intuition, which is not a part of reflective thinking.

A nurse educator is teaching a group of student nurses regarding depression, its pathophysiology, and the theories related to the disorder. What statements will the nurse instructor include about the theories of depression? Select all that apply. A) Sociocultural theory emphasizes the role that social stressors play in the development of depression. B) The sociocultural factor theory states that those who are depressed focus on negative messages in the environment and ignore positive experiences. C) The learning theory states that individuals learn to be depressed in response to a self-perception of a lack of control over their life experiences. D) The sociocultural factor theory suggests that all people have an inborn need for interpersonal relationships. E) The learning theory states that individuals with depression typically experience little success in achieving gratification and little positive reinforcement in coping with negative incidents

A, C, E) Among the theories of depression, the sociocultural theory emphasizes the role that social stressors play in the development of depression. Also, the learning theory states that individuals learn to be depressed in response to a self-perception of a lack of control over their life experiences. The learning theory also states that individuals with depression typically experience little success in achieving gratification and little positive reinforcement in coping with negative incidents. The cognitive theory states that those who are depressed focus on negative messages in the environment and ignore positive experiences. The interpersonal theory suggests all people have an inborn need for interpersonal relationships.

The nurse is instructing a client with an anxiety disorder on behavioral tools to help with coping. Which tools to help with coping should the nurse include in the teaching session? Select all that apply. A) Relaxation techniques B) Thought stopping C) Journaling D) Distraction E) Practicing yoga

A, C, E) Behavioral tools to help with coping include relaxation techniques, yoga, and journaling stressors and emotional responses and alternatives. Thought stopping and distraction are cognitive coping tools.

Which assessment findings indicate that a client is at increased risk for suicide? Select all that apply. A) Substance abuse B) Age 59 C) Plays golf twice a week D) Widowed for 6 months E) Recently started a new job

A, D) Of the assessment findings, the ones that would increase the client's risk for suicide are the recent loss of a spouse and substance abuse. For the risk of age, elderly male clients have the highest risk followed by adolescents and college students. Active employment and engagement in activities do not increase the risk for suicidal ideation or behavior.

The nurse is preparing to provide care to a group of clients. On which specific areas should the nurse focus in order to prioritize the clients' care needs? Select all that apply. A) Asking if any clients have complex issues B) Noting number of licensed staff assigned for the shift C) Noting time when the attending physicians make rounds D) Identifying clients with specific medication times E) Noting which clients have particular safety needs

A, D, E) Setting priorities for nursing care always begins with assessment. Assessment includes making observations and asking questions to gather information necessary to make decisions. Helpful assessment data include knowing which clients have complex care issues, if any clients have particular medication times, and if any clients have safety issues that should be addressed. The number of licensed staff is not as important as knowing the number of unlicensed assistive personnel to whom the nurse can delegate client care activities. The time when attending physicians make rounds is not usually part of the criteria when prioritizing client care needs.

A client is scheduled for electroconvulsive therapy (ECT) for the treatment of depression. Which instructions should the nurse include regarding this therapy? Select all that apply. A) You will need to remove all jewelry before beginning the therapy session. B) These treatments will cure the depression. C) Long-term memory loss often occurs after receiving ECT. D) The treatments are known to help some but not all people with depression. E) You will need to stop eating and drinking 4 hours prior to the therapy session.

A, D. E) The nurse should provide information to the client regarding ECT treatment for depression. ECT will not cure depression. Short-term memory loss is expected but long-term memory loss is not. The treatment is known to help some people who experiencing depression, but not all. The client must be NPO for 4 hours prior to receiving ECT treatment and must empty the bladder and remove contact lenses, jewelry, hairpins, and dentures before beginning the procedure.

Which assessment findings indicate to the nurse that a client is experiencing stress? Select all that apply. A) Chewing on a fingernail B) Checking cellular phone C) Reading a magazine D) Talking with others E) Tapping foot

A, E) The client is experiencing both behavioral (nail chewing) and physical (foot tapping) indications of stress. Reading a magazine, checking a phone, and talking with others are not indications of stress.

The nurse who uses clinical decision making to start CPR on a client is concerned about what other nursing concept? A) Cognition B) Perfusion C) Thermoregulation D) Acid-base balance

B) Cardiopulmonary resuscitation (CPR) is used to help restore circulation to major organs, which is a function of perfusion. Cognition, acid-base balance, and thermoregulation may also benefit from CPR, but this is not the nurse's primary concern when CPR is needed.

A home health nurse is precepting a new nurse during a routine wound care visit. The new nurse is assessing the client's wound and notes that it is showing signs and symptoms of infection. The client's spouse asks the new nurse how the wound looks. The new nurse responds by stating, "It looks fine," but the new nurse's face indicates a different story. When evaluating the new nurse, the preceptor should note a need to work on which aspect of communication? A) Credibility B) Congruence C) Timing D) Clarity and brevity

B) Congruence is adjusting one's tone of speech and facial expression to match one's spoken message. If a nurse's facial expression and words are not congruent, the client and family will often suspect that something is wrong. Credibility is the quality of being truthful, trustworthy, and reliable. Although the nurse's words in this situation may not be completely true, they do not necessarily reflect a larger problem with credibility. Timing means that a message is delivered when the client and family are capable of processing it fully and correctly. Clarity and brevity are characteristics involving preciseness and use of few words.

The nurse is preparing to triage victims of a train derailment who are being transported to the emergency department. Which victims would need immediate care? Select all that apply. A) Holding broken arm, sitting in a chair B) Respiratory rate of 8 and irregular C) Bleeding from fractured limb with a blood pressure of 78/40 mmHg D) Bleeding from superficial facial wounds and talking to family E) Walking with a slight limp, asking for something to drink

B) Emergent or immediate care is needed for life-threatening issues that require prompt treatment and care. Stabilization of the client's condition is critical. A respiratory rate of 8 and a blood pressure of 78/40 mmHg would be emergent. Urgent or delayed care is for serious health conditions in which a delay of treatment and care would not result in life-threatening situations. Holding a broken arm sitting in a chair and bleeding from superficial facial wounds while talking with family would be urgent. Nonurgent or minor issues do not require prompt care. Many of these clients can ambulate and are stable in their conditions. The client walking with a limp and asking for something to drink would be nonurgent.

The nurse is evaluating medication teaching for a client who recently started taking fluoxetine (Prozac) for anxiety. Which statement by the client indicates appropriate understanding of the information presented? A) "My medication will take 1 week to become effective." B) "My medication will take 4 weeks to become effective." C) "My medication will become effective immediately after I start taking it." D) "My medication will not begin to work for 12 weeks."

B) Fluoxetine (Prozac) is a selective serotonin reuptake inhibitor (SSRI). Although these drugs begin to alter brain chemistry after the first dose, their full effect requires a few weeks because a series of neurobiological changes must take place before SSRIs achieve efficacy.

A nurse is providing care to a woman who recently got married and would like to try to become pregnant. The woman has been on an antianxiety medication, paroxetine (Paxil), for the past year. The woman feels that she needs to continue receiving treatment for anxiety, especially if she gets pregnant. What information should the nurse provide regarding treatment options during pregnancy? A) The woman should consider switching to a different SSRI such as fluoxetine (Prozac). B) The woman should consider switching to cognitive-behavioral therapy (CBT) rather than medication. C) The woman should consider stopping all medications immediately. D) The woman should consider gradually decreasing medication until she finds out she is pregnant.

B) For women who are pregnant or trying to become pregnant, nonpharmacologic therapies such as CBT and relaxation techniques are the safest for reducing anxiety and preventing harm to the fetus. The woman should not stop all treatment immediately. When stopping medication, the medication should be tapered off, but this should continue even after she finds out she is pregnant, not until she finds out she is pregnant. If she changes to a different SSRI, she should change to sertraline (Zoloft), which does not have the same risk of birth defects as paroxetine or fluoxetine.

The nurse is assessing a 68-year-old client who appears disheveled. At previous appointments, the client was well kept with good hygiene practices. Today, the client's clothes do not match, the client's hair is unkempt, and the client has intense body odor. The nurse is concerned about this change in self-care. When conducting the assessment, what is the primary factor the nurse should consider? A) Whether the changes are due to a lack of understanding of technology B) Whether the changes are due to stress or dementia C) Whether the client is taking all medications as prescribed D) Whether the client is living independently

B) In older clients, changes in self-care habits are frequently in response to stress, but they could also be related to dementia and an inability to remember how to use basic grooming tools. The nurse needs to determine the cause of the self-care changes. A lack of understanding of technology and living independently could be causes of stress, and forgetting to take medications could be a sign of dementia, so these are secondary considerations after determining whether the changes are due to stress or dementia.

The nurse is caring for several clients who have plans to commit suicide. Which plan does the nurse identify as being most lethal? A) The individual who plans to use a mild overdose of pharmaceuticals B) The individual who plans to jump off a tall building C) The individual who plans to jump off a bridge into a river D) The individual who plans to slit across one wrist

B) Individuals who have only death in mind when attempting suicide will usually use a gun, hang themselves, or jump from a tall building, because these methods are more likely to case mortal injuries. Individuals who would like to die but really just want people to know how unhappy they are will often use less mortal means such as a mild overdose of pharmaceuticals or causing injuries that would be life threatening if no one found them in time.

Which of the following barriers to communication involves asking a client for information chiefly out of curiosity rather than with the intent to assist the client? A) Challenging B) Probing C) Testing D) Rejecting

B) Probing involves asking a client for information chiefly out of curiosity rather than out of a desire to assist the client. Probing often places clients in a defensive position and violates their privacy. Challenging refers to giving a response that makes clients prove their statement or point of view. Testing involves asking questions that make a client admit to something. Rejecting is refusing to discuss certain topics with a client. All four of these behaviors are barriers to effective nurse-client communication.

Which intervention is a primary prevention strategy for depression? A) Regular screening for depression B) Provide education about stress management C) Counseling clients about their risk for mood disorders D) Developing community-based mental health programs

B) Providing education about stress management is a primary prevention method for depression. Regular screening for depression and counseling clients about their risk for mood disorders are secondary prevention methods. Developing community-based mental health programs is a tertiary prevention method.

A client, who is experiencing anxiety, is trembling and complaining of dizziness and palpitations. The client is having a hard time following the nurse's instructions. Based on this data, which level of anxiety is the client likely experiencing? A) Panic B) Severe C) Moderate D) Mild

B) Severe anxiety can be associated with trembling, dizziness, palpitations, and difficulty following directions, among other signs and symptoms. Mild anxiety causes an increase in alertness and sensory perception. Moderate anxiety results in a reduction in awareness and increased restlessness and irritability. Panic is associated with dilated pupils, pallor, diaphoresis, and bizarre behavior.

The nurse is admitting a client to an inpatient psychiatric unit. The client is speaking wildly and is obviously very agitated. Which action by the nurse would be appropriate to calm the client? A) Placing the client in a private room, away from others B) Speaking to the client in a soft, calm tone C) Administering a prn medication to sedate the client D) Using short sentences when talking to the client

B) Speaking in a soft, calm manner is the first step in attempting to soothe an excited client. The nurse's tone may calm the excited client. Using short sentences is a useful approach, but in the case of an excited client, the nurse would attempt to calm the client first. Isolating the client may be necessary if the client's behavior escalates to violence, but that is not evident here and is not the first choice of action. Giving a sedative is the last resort and is used only if the client is threatening to hurt self or others.

Which intervention would help a client who is demonstrating stress about being hospitalized who is concerned about the needs of the children at home? A) Ask the client if there is anything that is needed once discharged to home. B) Ask the client if there is anyone who would be able to help with the family needs at home during recuperation. C) Find out if the children can be sent to a grandparent's home until the client fully recovers. D) Suggest the client be transferred to a long-term care facility to ensure a full recovery.

B) The nurse needs to focus on what can be done right away to help the client. The best way that the nurse can help this client is to ask if there is anyone who can help the client at home. Transferring the client to a long-term care facility will not help the client with the stress of caring for a family at home. Sending the children to a grandparent's home might not work if the children are in school and the grandparent lives far away. Asking the client if there is anything that is needed once discharged is not enough. The nurse needs to do something else.

The mother of a 12-year-old child with obsessive-compulsive disorder (OCD) tells the nurse that the child tends to get angry and throw a fit when the parents prevent him from performing compulsions in public. She tells the nurse that they don't have this problem at home because they just let him perform his rituals. The mother asks the nurse why he has these. What is the best response by the nurse? A) "It would be best if you don't take your child out in public until he can learn to control himself." B) "Rage attacks by children with OCD are often made worse if the parents accommodate the OCD behaviors." C) "The best way to prevent the rage attacks is to reinforce the OCD behaviors, especially when in public." D) "When he has rage attacks, you need to discipline him immediately and remove him from the area."

B) Studies indicate that children and adolescents are more likely than adults to suffer rage attacks in relation to their OCD, a phenomenon that is enhanced if their family accommodates or reinforces their OCD behaviors. Telling the mother about this link can help the parents see the need to change their actions, which may eventually help decrease the rage attacks in public. Not taking the child out in public, reinforcing the behaviors in public, or immediately disciplining the child will not be beneficial to the child.

A client complains about the stress of having to work long hours and missing daily exercise routines. Which response by the nurse is appropriate? A) "There are other ways to reduce stress, such as meditation." B) "Exercise helps reduce the impact of stress on the body and would be a good thing." C) "Drinking a small glass of wine each day does help reduce stress." D) "Maybe exercising, with all of the work, would be too much for your body anyway."

B) The client had been exercising but has not been recently because of additional work, which is causing stress. The nurse should encourage the client to resume daily exercise to reduce the impact of the stress on the body. The nurse should not reinforce the client's not exercising. Meditation might be beneficial, but because the client mentioned initially exercising and not meditating, this suggestion is not as appropriate in addressing the client's needs. The nurse should not suggest using alcohol to deal with stress.

A client states to the nurse that learning how to use the blood glucose machine will have to wait until holiday events are planned. Which cognitive indication of stress is the client demonstrating? A) Problem solving B) Suppression C) Self-control D) Cognitive structuring

B) The client is demonstrating suppression, which is the conscious process of denying unacceptable thoughts or emotions. The client is focusing on other needs and not the need to learn how to use the blood glucose machine. Problem solving involves thinking through a challenging situation, using specific steps to arrive at a solution. Cognitive structuring uses mental processes to make sense of environmental stimuli. Self-control is the ability to restrain oneself from acting on impulse or to act in such a way as to delay gratification.

A 68-year-old female client was recently diagnosed with depression and subclinical obsessive-compulsive symptoms. What does the nurse need to consider when planning care for this client? A) This client will not need treatment for the OCD symptoms because they are subclinical. B) This client may take longer to meet goals than a younger client with similar symptoms. C) This client will need to be assessed frequently for signs of dementia. D) This client may need a higher dose of medication than a younger client.

B) The comorbidity of obsessive-compulsive symptoms and other mental disorders may interfere with responsiveness to treatment and increase the time it takes for medications to be effective in older adults. The obsessive-compulsive symptoms do still need to be treated in the older adult, even though they are subclinical. Although older adult clients with multiple mental health disorders should be assessed for dementia, the assessments do not need to be performed more frequently than normal. Older adults often need lower doses of medication because of an increased risk of side effects.

A client with aspiration pneumonia is diaphoretic, pale, and taking gasping breaths. Which is the priority nursing action? A) Notify the healthcare provider. B) Complete a thorough cardiopulmonary assessment. C) Administer 10 L of oxygen per face mask. D) Reposition the client to help with breathing.

B) The first step in the nursing process is to complete an assessment of the client. The client is indeed experiencing difficulty, but the nurse needs to assess the extent of the need and the reason for the problem before taking action. The healthcare provider will ask the nurse to identify the reason for the problem and the extent of the problem. Administering oxygen could be dangerous to the client in some cases, such as if the client has chronic obstructive pulmonary disease. The client may have simply slipped down in the bed and need repositioning; on the other hand, the client may be in trouble, so making the decision to simply reposition the client without assessment could cause harm.

Which neurotransmitter change is frequently associated with suicide? A) Increase in serotonin B) Decrease in serotonin C) Increase in dopamine D) Decrease in dopamine

B) The most commonly studied neurotransmitter thought to be connected to suicide is serotonin. Suicidal individuals have been found to have decreased levels of serotonin, which can cause increased impulsivity and suicidal behavior.

The nurse is caring for a young adult client after a cervical biopsy. The client has expressed anxiety about the results. The healthcare provider peeks into the client's room and says, "The biopsy is negative." The nurse later finds the client sobbing. Which response by the nurse is most appropriate? A) "What did the healthcare provider tell you about the biopsy?" B) "You seem upset. Do you want to talk to me about the test results?" C) "Why are you crying after getting such good news?" D) "In this case, the term 'negative' is good!"

B) The nurse does not know specifically what the client is upset about and should ask the client an open-ended question so she can talk. The healthcare provider, in delivering important news to the client, should have taken time to sit with her and discuss the test results. In telling the client that the test was negative, the provider did not clarify what "negative" actually meant. Using medical jargon without explanation can lead to misinterpretation by the client. Asking the client why she is crying about good news does not allow the client to express concern regarding the results. Asking the client what the provider told her assumes that she is crying because of what the provider said and does not allow her to express her concern in an open-ended manner. Saying that the test results are good in this case assumes that the client has misunderstood the results, which may be true but does not allow the client to express her concerns.

The nurse is caring for a client with a new colostomy. The client has been taught how to perform colostomy care and has been successful with return demonstration to the staff. Although the client is able to perform care independently and has asked to do so, the charge nurse has instructed the nursing staff to continue performing colostomy care for this client. When addressing this issue directly with the charge nurse, which statement by a staff nurse is the most appropriate? A) "The client will change the apparatus whether you like it or not." B) "The client has been trained to change the apparatus and has expressed interest in performing this procedure independently." C) "You have no right to continue delegating this task to nurses when the client has been trained to change the apparatus." D) "I am going to tell the nurse manager that you won't allow the client to change the apparatus independently."

B) The nurse should make a clear, assertive statement saying that the client learned the procedure and wishes to execute it. Saying "You have no right . . ." is a challenge to the charge nurse and will only result in escalation of the argument. Telling the charge nurse that the nurse manager will be notified is a threat and inflames the situation. Insisting that the client will continue to change the apparatus will likely result in the charge nurse taking further action to prevent the client from performing self-care.

A female nurse is caring for a 21-year-old male client with a questionable gastrointestinal blockage. The healthcare provider prescribes an enema. Which reaction by the client would the nurse anticipate when planning care? A) "May I have a visitor in the room with me for support during the procedure?" B) "I would rather have my doctor perform this procedure." C) "I don't know what an enema is." D) "I am afraid of having an enema."

B) The nurse would anticipate that most young adult clients will be embarrassed by this procedure when the nurse and client are of different genders. When the client states that he would rather have his doctor perform the enema, he is probably motivated by embarrassment and acting on the assumption that the doctor is male. The nurse should approach the client beforehand to address the issue. Most clients would only experience annoyance, not fear, in relation to this procedure. Most clients in this age group would also be familiar with what an enema is, even if they have not had an enema themselves. The nurse would definitely not expect the client to request the presence of another individual in the room for this procedure.

What important fact should the nurse relay to the young adult who was just diagnosed with obsessive-compulsive disorder? A) Not acting on compulsions is the best cure. B) Treatment is essential to remission. C) Recognizing the obsessions as false will lessen their impact. D) The disorder will gradually get better over time.

B) Without treatment, the rate of remission of OCD is estimated to be low. Therefore, treatment is essential; the disorder will not usually get better over time without treatment. Many individuals with OCD recognize that their obsessive thoughts are false or unlikely to happen, but that does not lessen their impact. Individuals with OCD perform compulsions to lessen anxiety, and they are often unable to control whether or not they perform the actions.

The nurse is prioritizing care needed for a group of clients according to urgency. Which care should the nurse identify as being medium priority? Select all that apply. A) Instructing on changing ostomy appliance B) Performing passive range of motion every 4 hours C) Removing splints and providing complete skin care every 2 hours D) Administering 2 units of fresh frozen plasma E) Performing endotracheal suction

B, C) Medium-priority interventions are those that would not cause a life-threatening situation if performed later throughout the shift. These interventions include passive range of motion exercises and removing splints for skin care. Instructing on ostomy appliance changes is a low-priority intervention. Administering fresh frozen plasma and endotracheal suctioning would be considered critical interventions because if these interventions were not performed, the client's health status could quickly deteriorate.

A client with a 2-month-old child is experiencing insomnia, mood swings, and crying. Which interventions does the nurse anticipate being incorporated into a collaborative plan of care for the client experiencing postpartum depression? Select all that apply. A) Electroconvulsive therapy B) Psychosocial interventions C) Antidepressants D) Time management and exercise therapy E) Cognitive-behavioral therapy

B, C) The client is demonstrating signs of postpartum depression as evidenced by the mood swings, insomnia, and crying. Treatment for this disorder includes antidepressants and psychosocial interventions. Electroconvulsive therapy and cognitive-behavioral therapy would be indicated for some cases of depression but not for postpartum depression. Time management and exercise therapy would not be beneficial for a client experiencing postpartum depression.

The nurse is concerned that a client is demonstrating signs of obsessive-compulsive disorder. Which clinical manifestations and risk factors identified during the nursing assessment caused the nurse's concern? Select all that apply. A) Not making eye contact with the nurse B) Female age 25 C) Client checking the contents of a purse several times within minutes D) Client repeating the words "third floor" E) Client asking to use the bathroom in the middle of the assessment

B, C, D) Obsessive-compulsive disorder affects men and women equally; however, women often develop the disorder in adolescence or early adulthood. Checking and repeating are two common compulsions related to symmetry obsessions. Lack of eye contact and using the bathroom during an assessment are not manifestations of the disorder.

A client diagnosed with obsessive-compulsive disorder (OCD) is being admitted as an inpatient. The client is obsessed with thoughts of symmetry. Which compulsive behaviors does the nurse anticipate when performing the admission assessment? Select all that apply. A) The client repeatedly washes his hands. B) The client repeatedly taps both wrists on the bedside table. C) The client avoids shaking the nurse's hand D) The client begins counting the floor tiles. E) The client repeatedly cleans the top of the bedside table.

B, D) Repeatedly tapping both wrists on the bedside table and counting the floor tiles demonstrate common behaviors of a client whose obsession is symmetry. A client whose obsession is symmetry often demonstrates counting, ensuring orderliness of items, or fixation on maintaining symmetrical positioning of items, such as repeatedly tapping both wrists on the bedside table. On the other hand, a client whose obsession is cleaning typically demonstrates repetitive performance of decontamination practices, such as repetitive hand washing; avoidance of contamination, such as refusing to shake hands; or repetitive environmental cleaning, such as repeatedly cleaning the top of the bedside table.

A nurse is providing care for a client who has vocal cord damage and wants to implement strategies that will promote communication with this client. Which interventions would be appropriate? Select all that apply. A) Facing the client when speaking B) Having pen and paper on hand for the client C) Making sure that the language spoken is the client's dominant language D) Using a picture board to facilitate communication E) Employing an interpreter

B, D) The client who is nonverbal would respond best to use of a picture board or pen and paper. Because the client cannot communicate verbally, facing the client when talking, using an interpreter, or using the client's dominant language would not address the client's inability to communicate.

A client begins to vomit blood. The nurse immediately measures the blood pressure and prepares to insert a nasogastric tube while directing others to notify the healthcare provider and prepare to perform iced saline lavage. Which features of the Tanner Clinical Judgment Model did this nurse demonstrate? Select all that apply. A) Presencing B) Noticing C) Reflecting D) Interpreting E) Responding

B, D, E) The four features of the Tanner model include noticing, interpreting, responding, and reflecting. The nurse noticed that something was wrong with the client, interpreted the cues to gain an understanding of the situation, and responded by choosing the best course of action. The nurse has not yet reflected on the situation because she is still caring for the client. Presencing is being present for a client and engaging in face-to-face contact, which is not part of the Tanner model.

A mother says to the nurse, "I think my teenage son is showing signs of obsessive-compulsive disorder, just like his father." Which risk factors in the client's medical history would support this diagnosis? Select all that apply. A) Lives with parents B) Male gender C) Unemployed D) History of chronic illnesses E) Family history

B, E) Risk factors for obsessive-compulsive disorder include having a first-degree relative with the disorder and going through a major life stressor. Men develop the disorder earlier than women. Living with parents, being unemployed, or having a history of chronic illnesses are not risk factors for the disorder.

The nurse is prioritizing care for a client with several problems. List the order in which the nurse should address the client's needs. A) Bleeding through nasogastric tube B) Audible wheezes C) Not understanding how to complete the menu D) Requesting medication for arthritis pain E) Dyspnea F) Asking questions about teaching provided the other day

B, E, A, D, F, C Explanation: The nurse's priority should follow the ABCs, or airway, breathing, and circulation. Audible wheezes could indicate an obstruction and should be attended to first. Dyspnea indicates a problem with breathing and should be addressed next. Bleeding could impact circulation and should be addressed third. Pain control should be addressed fourth. Reinforcement of teaching can occur fifth, and helping with menu completion can occur last.

A malfunction in what system is thought to contribute to the development of obsessive-compulsive disorder? A) Frontal-subcortical circuit B) Hypothalamic-pituitary-adrenal axis C) Cortico-striato-thalamo-cortical circuit D) Microbiome-gut-brain axis

C) A malfunction in the cortico-striato-thalamo-cortical (CSTC) circuit in the brain is the possible cause for OCD. A malfunction in the other pathways contributes to other conditions or diseases, but they have not been linked to OCD.

A young adolescent client is in the hospital preparing for major surgery for the removal of a tumor on the kidney. The client's mother tells the nurse that she doesn't want her child to receive narcotics for postoperative pain. What is the nurse's best response? A) "Okay, I'll tell the healthcare provider not to order any. Are you sure you want to do this?" B) "The pain will be severe. Why don't we ask your child about this?" C) "Your child's pain will be severe after the surgery. Can you tell me why you feel this way?" D) "You do not have a choice of medication. Decisions involving pain relief are up to the healthcare providers."

C) As a client advocate, the nurse should defend the need for effective pain medication by using assertive communication. Thus, the best response is for the nurse to tell the mother the truth about the client's pain and then explore the mother's objection to the drugs. Acceding to the mother's request is submissive communication and would not be in the best interests of the child. Telling the mother she does not have a choice is aggressive and untrue. Putting the child in the middle of the discussion is a divisive maneuver.

A client is prescribed fluoxetine (Prozac) for treatment of obsessive-compulsive disorder. During the latest office visit, the client washes the hands while counting to 10 and repeats the process every 5 minutes. Which is the priority assessment for the nurse to complete for this client? A) The amount of medication the client is taking B) Side effects from the medication the client is experiencing C) Whether the client is taking the medication as prescribed D) Foods that may be interacting with the client's medication

C) Fluoxetine (Prozac) is one medication prescribed for the treatment of obsessive-compulsive disorder. Because the client is demonstrating continuing signs of the disorder, the nurse should assess if the client is taking the medication as prescribed. The client would have other signs and symptoms if taking too much medication. There are no specific foods to avoid when taking this medication. Continuing symptoms of obsessive-compulsive disorder is not a side effect of the medication.

After a mammogram, a client is told that she needs a fine needle aspirate of a breast mass. Which action by the client demonstrates engagement in a primary appraisal of the stressful situation? A) Holding her breath while the nurse is talking B) Sitting in the dressing room crying C) Asking the nurse if she has cancer D) Scheduling the procedure in 6 weeks, which is the earliest possible appointment

C) In primary appraisal, the client assesses the potential for benefit, harm, loss, threat, or challenge in a situation. The client asking the nurse if she has cancer is engaging in a primary appraisal. The client holding her breath while the nurse is talking is evaluating coping resources and options. This is a secondary appraisal. The client who sits in the dressing room and cries is applying a coping resource. This is coping. The client who schedules the procedure at the earliest possible appointment is engaging in reappraisal, which is an ongoing reinterpretation of the situation based on new information.

A nurse on the behavioral health unit is caring for a client diagnosed with depression who just lost a spouse in a motor vehicle crash. The client states to the nurse, "My wife would not have wanted to live if she were disabled." Based on this statement, which defense mechanism is the client using? A) Identification B) Denial C) Intellectualization D) Displacement

C) Intellectualization is a mechanism by which an emotional response that normally would accompany an uncomfortable or painful incident is evaded by the use of rational explanations that remove from the incident any personal significance and feelings. Identification is an attempt to manage anxiety by imitating the behavior of someone feared or respected. Denial is an attempt to screen or ignore unacceptable realities by refusing to acknowledge them. Displacement is the transferring or discharging of emotional reactions from one object or individual to another object or individual.

The nurse is caring for an older school-age client who is sleeping when the menu choices for dinner are brought to the room. Which intervention should the nurse use to meet the dietary needs of this client? A) Wake the child to choose a meal for dinner. B) Order chicken nuggets because most children like this meal. C) Ask the dietary worker to come back later. D) Ask the parents to bring dinner from home for the client.

C) Involving children in their own care increases cooperation and decreases anxiety. The nurse would ask the dietary worker to come back later to increase the child's involvement in his own care and to avoid disturbing the client or choosing a meal the client won't eat. If the parents are present, the nurse might ask them if they are comfortable making choices for the child, but asking them to bring food in is inappropriate.

A nurse working on a psychiatric unit is caring for a client who has been diagnosed with major depressive disorder (MDD). Upon assessment of the client, which clinical manifestations does the nurse recognize as consistent with this diagnosis? A) Depressed mood or loss of interest occasionally for at least 1 week B) Depressed mood sporadically for at least 2 years C) Restlessness, fatigue, suicidal ideation, feelings of guilt D) Anxiety, change in appetite, grief, altered nutrition

C) MDD is diagnosed when the client experiences either depressed mood or loss of interest most of the day, almost every day, for at least 2 weeks. The depression must be accompanied by at least four symptoms, including sleep disturbance, fatigue, feelings of guilt or worthlessness, restlessness or psychomotor agitation, and suicidal ideation or attempt.

A novice nurse is planning care for an older adult client with a wound infection and systemic blood infection. The nurse completes the plan of care and decides to complete which action to enhance the skill of critical thinking? A) Discuss the plan with the physician. B) Request that the client review the plan. C) Request a review of the plan with the nurse's preceptor. D) Place the plan on the client's chart.

C) Part of the critical thinking process is inquiry, or searching for knowledge or facts. One way the novice nurse can implement inquiry is to discuss the care plan with a more experienced nurse to determine if additional or better interventions need to be added to the care plan. The nurse would seek the opinion of another nurse, not the physician or the client, as the goal is to enhance the nurse's ability to use the nursing process. The nurse could put the plan on the chart, but the new nurse would learn more about critical thinking by requesting assistance from a more experienced nurse.

A client, who was recently laid off from work, is scheduled for a biopsy to evaluate a site for malignancy. When planning this client's care, which does the nurse include? A) Reasons to delay the biopsy B) Medicate around the clock for pain C) Interventions to address anxiety D) Social services to aid with financial planning

C) Risk factors for anxiety disorders include multiple stressors such as an illness occurring with a change in employment. The nurse should plan interventions to address anxiety. Social services may or may not be needed for the client's financial planning. Delaying the biopsy will not help reduce anxiety. There is no evidence to suggest the client is experiencing pain.

Which of the following statements on the part of the nurse is an example of the communication barrier known as testing? A) "Most people have little to no pain after this type of procedure." B) "Tell me when and why you started smoking marijuana." C) "Do you think you're the only client on the unit right now?" D) "How are you still in pain after receiving both doses of medication?"

C) Testing involves asking questions that make the patient admit to something. These responses permit the patient only limited answers and often meet the nurse's need rather than the patient's. Of the options listed here, "Do you think you're the only client on the unit right now?" best meets this definition. Telling the client that most people have little or no pain after a procedure is an example of stereotyping. Asking about why a client started using marijuana may be an example of probing. Questioning how the client is still in pain after medication administration is an example of challenging, or making clients prove their statements or point of view.

Which child would the nurse recognize as being at the highest risk of experiencing toxic stress? A) A 15-year-old adolescent who is slightly overweight and didn't make the football team; he regularly gets teased for his weight at school. B) A 2-week-old infant who was born at 31 weeks' gestation and has been in the neonatal intensive care unit (NICU) for the entire 2 weeks; the child's parents are at the hospital as often as possible. C) A 12-year-old child whose father recently died and whose mother works three part-time jobs; this child is expected to care for two younger siblings after school. D) A 4-year-old child who attends preschool or daycare each day while the parents work; the child displays signs of mild separation anxiety.

C) The 4-year-old child and 15-year-old adolescent are experiencing normative stressors, which do not usually lead to toxic stress. The 2-week-old infant may be experiencing non-normative stress, but the infant is receiving appropriate adult support and is likely too young to be cognitively aware of stressors. The 12-year-old child has experienced a non-normative stressor in the death of the father, and the child does not have adequate adult support. This places the child at high risk for toxic stress.

The nurse is planning care for an adult client demonstrating symptoms of depression. Which assessment technique is most appropriate? A) More time talking with the client B) Ask family members about the client's demeanor C) Beck Depression Inventory D) Mood Disorder Questionnaire

C) The Beck Depression Inventory is a series of 21 questions that the client answers in order to self-rate the level of depression. It takes approximately 10 minutes for the client to complete. The nurse can use it to help with the assessment of this client. The Mood Disorder Questionnaire is used to screen adolescents for bipolar disorder, rather than general depression. The nurse should not ask family members to talk about the client. Assessment of clients with depression is often done in 15- to 20-minute increments because the client usually does not have the energy to talk much longer. For that reason, the nurse should not plan more time with the client to complete the assessment.

The nurse is caring for a client who is having difficulty understanding the dressing changes that need to be completed in the home as part of postdischarge wound care. The client asks the nurse to demonstrate the procedure again and allow the client's spouse to perform the procedure while the nurse watches. What is the most likely outcome of this assertive request by the client? A) A slightly increased chance that the wound will become infected due to exposure during dressing changes B) Less compassionate care for the client due to the nurse's irritation by the request C) A greater likelihood that the wound will heal appropriately D) A guarantee that the spouse will change the dressings correctly

C) The client used assertive communication to ensure that the dressing changes would be performed correctly, which will likely result in appropriate healing of the client's wound. No information is provided about the nurse's response to the request, and even if the nurse is irritated, these feelings should not affect the quality of care. There is no guarantee that the client's spouse will always perform the task correctly, because humans make mistakes. Infection of a wound that is dressed correctly is not the likely result of this request.

A nurse is evaluating the plan of care for a client diagnosed with obsessive-compulsive disorder (OCD). Which client statement indicates a positive outcome for the plan of care? A) "Instead of washing my hands several times a day I use hand sanitizer several times a day." B) "I am still hand washing frequently, and even though it is less than before I am a failure." C) "I am still hand washing frequently but it is less often than before. I think I am improving." D) "I don't know why I can't wash my hands several times a day; I have nothing else to do anyway."

C) The client who acknowledges improvement when washing hands less frequently recognizes that continued obsessive-compulsive behaviors are not an indication of treatment failure and that reductions in behavior signify positive progress. The client who has substituted the ritualistic use of hand sanitizer for the ritualistic hand washing has not demonstrated adequate coping skills to control anxiety related to absence of ritualistic compulsive behaviors. The client who sees nothing wrong with washing hands several times a day does not recognize that the ritualistic hand washing is a problem.

The nurse is assigned two clients. One client needs postoperative teaching in preparation for discharge, and the other client with pneumonia has a PaCO2 of 85. Why does the nurse decide to see the client with pneumonia first? A) The nurse can delegate postoperative teaching to unlicensed assistive personnel (UAP). B) The client with pneumonia needs more care than the client needing postoperative teaching. C) The client with pneumonia may be experiencing respiratory distress. D) The room of the client with pneumonia is closer than that of the client needing postoperative teaching.

C) The client with a PaCO2 of 85 could be in serious trouble. The nurse would decide to assess that client first in order to prevent dire consequences for the client. The client with pneumonia probably needs more care than the client preparing for discharge, but the reason for the decision is based on a potentially critical need by the client with pneumonia. The nurse cannot delegate discharge teaching to a UAP; even if delegation were permitted, the nurse would see the client with a high PaCO2 as being the greater priority. Placement of the client's room can be a decision that is made when considering time management issues; however, the physiologic needs of the clients are the first consideration of the nurse.

The nurse is providing care for a client who is about to be discharged. The nurse is discussing the discharge orders with the client's primary healthcare provider. Which statement by the nurse is an appropriate example of using assertive communication? A) "Can we talk about this client prior to discharge?" B) "That new medication you prescribed for the client is ineffective." C) "I am worried about the client's blood pressure. It remains high even with the new medication." D) "Excuse me, Doctor, I think you need to do something about the client's blood pressure."

C) The nurse who expresses concern because the client's blood pressure remains high even with new medication is being assertive, clear, and concise. Stating that the new medication is ineffective could be interpreted as an inflammatory remark by the provider. Asking whether the provider can talk about a client does not give the provider enough information. Saying the provider must do something about the client's blood pressure may cause the provider to become defensive.

The nurse is caring for an older adult client with decreased energy who needs to get up to prevent the development of pressure ulcers. The client is unable to ambulate and wants to be alone. What should the nurse do? A) Notify the healthcare provider of the client's noncompliance. B) Leave the client alone until ready to get out of bed. C) Gain knowledge about the client from family to gain compliance. D) Proceed to get help to get the client out of bed.

C) The nurse would use knowledge and creativity to think critically about getting the client to cooperate with the medical regimen. For example, the nurse would ask the family to become involved in order to gain compliance. It is not an option to leave the client alone and not address some method of ensuring intact skin. The compliance of the client rests with the nurse, not the physician. It would not be a good idea to force the client against the client's will; the nurse would use critical thinking to find another way to meet the goal.

A nurse is providing discharge instructions to a client recently diagnosed with obsessive-compulsive disorder (OCD) and prescribed fluvoxamine (Luvox). Which statement made by the client indicates to the nurse that the client understands the instructions? A) "I am glad the physician chose this medication because it does not have any side effects." B) "I should continue taking this medication and in 1-2 years I can stop taking it." C) "I should continue taking this medication and in 1-2 years my physician may taper me off gradually." D) "Even though I don't think this medication is for my OCD, I will take it because the physician wants me to."

C) This statement indicates that the client understands instruction regarding this medication. A client should continue taking fluvoxamine (Luvox) for 1—2 years, at which time a physician may begin gradually tapering, while observing the client for symptom exacerbation. Fluvoxamine (Luvox) does have side effects; however, it has fewer side effects than clomipramine and is recommended for the first medication trial. Fluvoxamine (Luvox) is approved by the U.S. Food and Drug Administration (FDA) for treatment of OCD.

An adolescent client hospitalized with asphyxiation following a suicide attempt tells the nurse, "I know other kids have the same problems I do, but I just wanted to make it stop." Which action by the nurse is the most appropriate? A) Discuss the client's attendance at school and what activities are enjoyed. B) Suggest the client listen to music and read a light novel to reduce stress. C) Ask if the client would like to talk about stressors and problems. D) Ask what is so devastating that the client needed to commit suicide.

C) Those who attempt suicide are overcome by and overwhelmed with stressors in their lives. The nurse should ask the client to talk about her stressors and problems. The nurse should not ask the client about what caused the attempt at suicide. The nurse should also not try to distract the client by asking about school and activities. Suggesting the client read and listen to music may not be sufficient to reduce the client's stress.

A client is experiencing symptoms of depression. Which laboratory or diagnostic test would be the priority to determine if depression is being caused by another health problem? A) Electrocardiogram B) MRI of the brain C) Thyroid function tests D) Cerebral angiogram

C) Thyroid function tests would be performed because thyroid disorders may mimic depression or hypomania. Cerebral MRI, cerebral angiogram, and electrocardiogram tests are not able to distinguish between mood disorders and physical disorders.

The nurse is starting preoperative teaching when the client receives a phone call. When the call ends and the nurse resumes teaching, the client is visibly upset and begins to cry. Which therapeutic initial response by the nurse is appropriate? A) "You can deal with whatever is upsetting you once we have finished." B) "It's very important to focus on this teaching so that you will recover quickly after surgery." C) "I can see that phone call has upset you. Let's talk about why you are upset before we move on with teaching." D) "What can you do to solve the problem?"

C) When clients are anxious and upset, they have difficulty focusing, especially on new information, and their anxiety becomes a barrier to effective communication. The nurse needs to prioritize the client's presenting need to express concerns and then engage in teaching at a later time, when the client is able to comprehend. Telling the client to focus will not relieve the anxiety of the imminent issue. Because the nurse is unaware of the issue and is not knowledgeable of the client's situation, it is false reassurance to tell the client that the matter can be handled after the teaching is completed. This response also belittles the client's independence and self-efficacy, as does recommending problem solving.

The nurse is prioritizing care activities that are to be completed for a group of clients. From highest to lowest priority, list the order in which the nurse should complete the listed activities. A) Measure blood pressure before administering antihypertensive medication. B) Request dietary consult for gluten-free diet. C) Remove an intravenous access device infusing chemotherapy. D) Change a dressing on an arm wound. E) Call a family member to bring in shoes. F) Ambulate to the bathroom using a walker.

C, A, D, F, B, E Explanation: The highest-priority action would be to remove an intravenous access device infusing chemotherapy because this could lead to significant tissue damage. Measuring blood pressure before administering antihypertensive medication is the next priority because this could be a timed intervention. Changing a dressing on an arm wound can occur anytime throughout the shift. Ambulating using a walker to the bathroom can be delegated to unlicensed assistive personnel. Requesting a dietary consult for gluten-free diet can also be done at any time and does not have a specific degree of urgency. Contacting family to bring in shoes can be completed at the end of the shift or delayed to the next shift.

A nurse is conducting an admission assessment on a client admitted for thoughts of suicide. Which assessment findings would indicate that the client is at a high level risk of suicide? Select all that apply. A) Displays mild depression. B) Shows curiosity about death. C) Has access to a gun at home. D) Admits planning to end his or her life. E) Discusses a plan to end his or her life in detail.

C, D, E) A client who has access to a gun at home, admits planning to end his or her life, and discusses a plan to end his or her life in detail is at high risk for suicide. Displaying mild depression and showing curiosity about death indicate low-level risk for suicide.

The nurse is prioritizing care for a client based on nursing diagnoses. If following Maslow's hierarchy of needs, list the order in which the nurse should provide care to the client. A) Fatigue B) Anxiety C) Alteration in Perfusion D) Self-Care Deficit E) Deficient Knowledge F) Diarrhea

C, F, D, B, A, E Explanation: High-priority nursing diagnoses should be addressed first. This means that Alteration in Perfusion would be the first priority. Medium-priority diagnoses should be addressed next, in order of the impact on physiologic processes. Diarrhea would be a priority over Self-Care Deficit; however, Self-Care Deficit would be a priority over Anxiety. Low-priority diagnoses would be addressed last, again in order of physiologic impact. Fatigue would be a priority over Deficient Knowledge.

A client with congestive heart failure (CHF) is having difficulty breathing. Before leaving the room, the nurse ensures the client has an overbed table to lean on when awake if needed to ease breathing. Which technique did the nurse use to make this decision? A) Delegating a task B) Priority setting C) Conflict resolution D) Critical thinking

D) After assessing the client, the nurse sets goals for and with the client. To arrive at the goal, the nurse uses critical thinking to make the decision to provide the client with optimum ability to breathe. Delegating involves giving the task to another team member. There is no conflict in this decision. Conflict resolution usually involves a compromise that affects two sides that are in disagreement. Priority setting involves deciding which task to perform first.

A 76-year-old man was recently diagnosed with Alzheimer disease. His wife passed away 6 months ago due to metastatic breast cancer. The client states that he doesn't sleep well, often forgets to eat because he doesn't feel hungry, and he just doesn't get involved in social functions anymore because his kids don't want him to drive. He states that he feels isolated and lonely. What diagnosis should the nurse include as the highest priority in this client's plan of care? A) Ineffective Activity Planning B) Grieving C) Risk for Loneliness D) Risk for Suicide

D) Although all of these diagnoses may be included in the client's plan of care, the diagnosis with the highest priority is risk for suicide. Risk factors for suicide that this client displays are age over 65 years, male client, symptoms of depression, death of a close loved one, and diagnosis of a terminal illness. Older adults have a higher rate of success when attempting suicide because they tend to use more lethal means. Therefore, risk for suicide should be a high-priority diagnosis for this client.

The nurse is caring for a client with a chronic health condition. Which condition should the nurse identify as a common complication associated with reduced role function? A) Osteoporosis B) Congestive heart failure C) Diabetes D) Depression

D) Depression is one of the most common complications arising with chronic disease. There is not enough information to determine if the client will develop osteoporosis, congestive heart failure, or diabetes.

Which instruction by the nurse to a client prescribed diazepam (Valium) for anxiety and stress is appropriate? A) "This medication will be good to take for a long time." B) "Take this medication every time feelings of stress become overwhelming." C) "This medication works best if taken with a meal." D) "This medication is good to use for the short term only."

D) Diazepam (Valium) is a benzodiazepine that is typically used for short-term treatment during an acute phase of an anxiety disorder. It may be effective in quickly lowering the severity of a client's anxiety but is generally not recommended for use beyond a few weeks because of its addictive properties. The nurse should instruct the client that the medication is good to use for the short term only. There is no indication that this medication needs to be taken with a meal. Instructing the client to take the medication every time feelings of stress become overwhelming could lead to an overdose and should not be done.

A postoperative client prescribed pain medication every 4 to 6 hours is requesting medication every 6 hours. At 4 hours the client's pain level is 8 on a rating scale of 1 to 10. The nurse decides to give the pain medication now. What does this nurse's action exemplify? A) Meeting a client goal B) Time management skills C) Prioritizing the client's care D) Responding to a change in the client's condition

D) Each client has a plan of care, but it is the nurse who constantly evaluates the client for changes that the nurse responds to, if needed. Prioritizing involves choosing tasks in order of importance. Time management is completing the assigned tasks in the given time frame by organizing and using efficiency. The goal has not been met if the client is experiencing pain.

A nurse is teaching a client about a dressing change that should be done three times per day. The client is from a culture that is "present oriented." Based on this data, at which times should the nurse tell the client to perform the dressing changes? A) At whatever times the client selects, as long as they are 8 hours apart B) At 9 a.m., 3 p.m., and 9 p.m. C) At whatever times the client selects, as long as the dressing is changed three times each day D) After breakfast, lunch, and dinner

D) For clients who are "present oriented," it is important to avoid fixed schedules. The nurse can offer a time range for activities and treatments, such as in the morning or after breakfast, and in the evening or before going to bed. Relating the dressing changes to regular daily activities would be a good approach for a client who is not focused on times of the day, such as 9 a.m., 3 p.m., and 9 p.m. It is not necessary for the dressing changes to be exactly 8 hours apart. Leaving it up to the client to change the dressing at any time as long as it is changed three times a day does not allow for any regularity in the dressing changes.

A nursing instructor is evaluating a nursing student's knowledge regarding a client with suicidal thoughts. Which statement made by the student demonstrates an understanding regarding assessing a client with suicidal thoughts? A) "I should attempt to make light of the circumstances." B) "I should be indirect and respectful." C) "I should not talk about suicide directly." D) "I should directly acknowledge the situation."

D) Individuals who are experiencing suicidal ideation will respect direct acknowledgment of the situation as opposed to a restrained approach. Nurses recognize the severity and finality of the client's decision and do not attempt to make light of the circumstances. Nurses working with clients who are considering suicide, or who have recently attempted suicide, should be direct but respectful when evaluating the client and asking questions. It is a common misconception that talking about suicide directly could cause the client to act in a suicidal manner, but this is not true.

A client is recently prescribed risperidone (Risperdal) by the healthcare provider. Which would be a priority nursing consideration for this client? A) Assess blood pressure and heart rate. B) Monitor for increased agitation. C) Assess for drowsiness. D) Monitor for neuroleptic malignant syndrome.

D) Monitoring for neuroleptic malignant syndrome is a priority nursing consideration for a client taking risperidone (Risperdal). The nurse must monitor for signs and symptoms of neuroleptic malignant syndrome and tardive dyskinesia and immediately report signs and symptoms of these conditions. Monitoring for increased agitation and assessing for drowsiness are nursing considerations for clients taking Risperdal, but they are not the priority diagnosis. Assessing blood pressure and heart rate would be a priority nursing consideration for the client taking Inderal.

What is one of the primary reasons that it is important for nurses to prioritize care? A) Nurses need to plan how to accomplish all activities within one shift. B) Nurses can accomplish more if they perform the easiest or fastest interventions first. C) Nurses should perform interventions related to client preferences early in the shift. D) Nurses only have a limited amount of time to perform nursing interventions.

D) Nurses only have a limited amount of time to perform nursing interventions. By prioritizing care, nurses can ensure that high-priority interventions are completed first, followed by medium-priority and then low-priority interventions as time allows. It will likely not be possible for a nurse to plan how to accomplish all activities within one shift. Nurses often accomplish less and are more stressed if they perform the easiest or fastest interventions first. Nurses should consider client preferences for all interventions regardless of the time the intervention is completed.

The home care nurse observes a client scrubbing areas throughout the house over and over, especially areas where the family gathers. Prior to planning care for this client, which must the nurse assess? A) If the client is forgetful B) If the client does not clean thoroughly C) How frequently the client cleans the house D) The impact of symptoms on the family system

D) Obsessive-compulsive disorder impacts the family system, especially with impaired role function. How frequently or thoroughly the client cleans the house may be important to assess, but they are not the most important. Forgetfulness is not a component of obsessive-compulsive disorder.

A client states that he often wonders if everyone would be better off if he were dead. What does the nurse identify this as? A) A suicide attempt B) Suicide planning C) A suicide threat D) Suicidal ideation

D) Suicidal ideation is having thoughts of harming or killing oneself. A suicide threat is more serious than a casual statement of suicidal intent and is accompanied by other behavior changes. A suicide attempt is a nonfatal, self-inflicted destructive act with explicit or inferred intent to die. A suicide plan is a decision to commit suicide and an identified method.

The nurse is providing care to a client who is diagnosed with obsessive-compulsive disorder. Which nursing intervention is most appropriate when providing care to this client? A) Confront the client and ask what purpose the behavior serves. B) Tell the client that the behavior is unacceptable and must end. C) Interrupt the ritualistic behavior when observed. D) Discuss the need to incorporate the behavior with other hospital routines.

D) The client with obsessive-compulsive behavior will not be able to perform the behavior at will, so the nurse needs to discuss the need to incorporate the behavior with other hospital routines. The nurse should not interrupt the behavior, as this will cause the client to start over from the beginning. The nurse should also not confront the client and ask what purpose it serves, as the client might be embarrassed about the behavior. Telling the client that the behavior is unacceptable and must end also will not help the client with the behavior.

A nurse receives a shift report and is preparing to care for clients assigned on a medical-surgical unit. Which client should the nurse plan to assess first? A) The client who needs assistance with activities of daily living B) The client who needs help ambulating to the bathroom C) The client with a pain rating of 3/10 D) The client experiencing shortness of breath

D) The nurse begins by assessing the client who is at the greatest risk, who in this case is the client having trouble breathing. The risk for the client with mild pain is not as severe as that for the client with dyspnea. The nurse can delegate the ambulation of a client to a nursing assistive individual. The nurse can also delegate assisting a client who needs help with a bath.

A client being treated for severe depression reports feeling better and having more energy. Which is a priority nursing diagnosis for the client at this time? A) Social Isolation B) Hopelessness C) Situational Low Self-Esteem D) Risk for Self-Directed Violence

D) The one risk that occurs with successful treatment of a client with depression is that once the depression begins to resolve, the underlying thought of suicide could prevail. With treatment, the client may begin to have more energy to make a plan regarding suicide. The nurse should further assess this client's statement about making plans. The client is not demonstrating low self-esteem, hopelessness, or social isolation.

The nurse is caring for a client recovering from a suicide attempt. Which client statement indicates to the nurse that the risk of suicide has diminished? A) "I am not looking forward to going home with my parents." B) "I now know that threatening suicide will help me get what I want from my parents." C) "Even though I failed this time, I lived to think about it again." D) "I am looking forward to going to school and seeing my friends."

D) The statement "I am looking forward to going to school and seeing my friends" indicates hope for the future, which is a desired outcome of care for a client who is at risk for suicide. The other statements indicate that the client may need further intervention either before discharge or once at home.


Kaugnay na mga set ng pag-aaral

Graphs and equations of motion - 3.4

View Set

Student Government Association Constitution Trivia

View Set

College Credit English Semester 2 Exam

View Set

CHAPTER 10 - ANKLE AND FOOT JOINTS

View Set